Category Archives: Thi chọn đội tuyển

Đề thi và đáp án kì thi chọn đội tuyển thi Quốc gia trường Phổ thông Năng khiếu năm học 2010 – 2011

ĐỀ THI

Ngày thi thứ nhất

Bài 1. Giải hệ phương trình sau:

$\quad\quad\quad\quad\quad\quad\quad\quad\quad\quad \left\{\begin{array}{l}\frac{5(x+y)}{x+y+6 x y}+\frac{6(x+z)}{x+z+5 x z}=4 \\ \frac{6(y+z)}{z+y+4 z y}+\frac{4(x+y)}{x+y+6 x y}=5 \\ \frac{4(x+z)}{x+y+5 x z}+\frac{5(y+z)}{y+z+4 y z}=6\end{array}\right.$

Bài 2. Tìm tất cả các hàm số $f: \mathbb{R} \rightarrow \mathbb{R}$ thỏa mãn:

$\quad\quad\quad\quad\quad\quad\quad\quad\quad\quad  f(|x|+y+f(y+f(y)))=3 y+|f(x)|, \forall x, y \in \mathbb{R}$

Bài 3. Cho $p$ là số nguyên tố lẻ và $n=2 p+r$ với $r \in{0,1,2, \ldots, p-1}$. Đặt $X={1,2, \ldots, n}$. Ánh xạ $f: X \rightarrow X$ được gọi là có tính chất $\mathcal{P}$ nếu $f$ không phải là ánh xạ đồng nhất và $f(f(\ldots(f(k)) \ldots)$ ) $=k$ (ánh xạ hợp $p$ lần) với mọi $k \in X$.

Đặt $A_f={k \in X \mid f(k)=k}$.

a) Chứng minh rằng nếu $f$ có tính chất $\mathcal{P}$ thì $\left|A_f\right| \equiv r(\bmod p)$.

b) Gọi $d$ là số các ánh xạ có tính chất $\mathcal{P}$. Chứng minh rằng $d$ không là ước của $n$ !.

(Kí hiệu $|A|$ chỉ số lượng các phần tử của tập hợp $A$.)

Bài 4. Cho tam giác $A B C$ nội tiếp đường tròn $(O)$ có $A$ cố định và $B, C$ thay đổi trên $(O)$ sao cho $B C$ luôn song song với một đường thẳng cố định. Các tiếp tuyến của $(O)$ tại $B$ và $C$ cắt nhau tại $K$. Gọi $M$ là trung điểm của $B C, N$ là giao điểm của $A M$ với $(O)$. Chứng minh rằng đường thẳng $K N$ luôn đi qua một điểm cố định.

Ngày thi thứ hai

Bài 5. Chứng minh rằng nếu $a, b, c$ là độ dài ba cạnh của một tam giác thì:

$\quad\quad\quad\quad\quad\quad\quad\quad\quad\quad (2 a+2 b-c)(2 b+2 c-a)(2 c+2 a-b)>25 a b c$

Bài 6. Cho dãy số $\left(u_n\right)$ thoả mãn $u_1=\sqrt{2}$ và $u_{n+1}=\frac{2 u_n^2+5 u_n+5}{2 u_n+4}, \forall n \geq 1$. Tìm $\lim \frac{u_n^2-3 u_n+5}{3 n^2+4 n-1}$.

Bài 7. Xét số tự nhiên $n>1$. Bắt đầu từ bộ số $1,2, \ldots, 2 n-1,2 n$, ta thực hiện phép biến đổi sau: Chọn hai số $a, b$ sao cho $a-b>1$, xoá hai số này và thay thế bởi hai số $a-1, b+1$. Với bộ số mới, ta lại tiếp tục thực hiện phép biến đổi tương tự’

a) Chứng minh rằng ta sẽ đạt đến trạng thái dừng, tức là không thể tiếp tục thực hiện phép biến đổi như vậy được nữa.

b) Gọi $k$ là số lần phép biến đổi cần thực hiện để đạt đến trạng thái dừng. Tìm giá trị nhỏ nhất và lớn nhất của $k$.

Bài 8. Cho đường tròn $\left(\gamma_1\right)$ đường kính $A B$ và đường tròn $\left(\gamma_2\right)$ tâm $A$ cắt $\left(\gamma_1\right)$ tại $C, D$. Điểm $M$ thay đổi trên cung $C D$ (nằm bên trong $\left(\gamma_1\right)$ ) của $\left(\gamma_2\right)$. Gọi $B M$ cắt $\left(\gamma_1\right)$ tại $N$ khác $M$ và $B$. Tìm giá trị nhỏ nhất của $\frac{N D+N C}{M N}$.

 

LỜI GIẢI

Ngày thi thứ nhất

Bài 1. Giải hệ phương trình

$\quad\quad\quad\quad\quad\quad\quad\quad\quad\quad \left\{\begin{array}{l}\frac{5(x+y)}{x+y+6 x y}+\frac{6(x+z)}{x+z+5 x z}=4 \\ \frac{6(y+z)}{z+y+4 z y}+\frac{4(x+y)}{x+y+6 x y}=5 \\ \frac{4(x+z)}{x+y+5 x z}+\frac{5(y+z)}{y+z+4 y z}=6\end{array} .\right.$

Lời giải. Đặt $u=\frac{x+y}{x+y+6 x y}, v=\frac{y+z}{y+z+4 y z}, w=\frac{z+x}{z+x+5 z x}$ thì ta có hệ

$\quad\quad\quad\quad\quad\quad\quad\quad\quad\quad \left\{\begin{array} { l }{ 5 u + 6 w = 4 } \\ { 6 v + 4 u = 5 } \\ { 4 w + 5 v = 6 }\end{array} \Leftrightarrow \left\{\begin{array}{l}8 u=1 \\ 4 v=3 \\ 16 w=9\end{array} .\right.\right.$

Suy ra

$\quad\quad\quad\quad\quad\quad\quad\quad\quad\quad \left\{\begin{array} { l }{ 7 ( x + y ) = 6 x y } \\ { 3 ( y + z ) = 1 2 y z } \\ { 7 ( z + x ) = 4 5 z x }\end{array} \Leftrightarrow \left\{\begin{array}{l}a+b=\frac{6}{7} \\ b+c=12 \\ c+a=\frac{45}{7}\end{array}\right.\right.$

trong đó $a=\frac{1}{x}, b=\frac{1}{y}, c=\frac{1}{z}$. Giải hệ trên, ta thu được $a=-\frac{33}{14}, b=\frac{45}{14}, c=\frac{123}{14}$ nên $(x, y, z)=\left(-\frac{14}{33}, \frac{14}{45}, \frac{14}{123}\right)$.

Bài 2. Tìm tất cả các hàm số $f: \mathbb{R} \rightarrow \mathbb{R}$ thỏa mãn:

$\quad\quad\quad\quad\quad\quad f(|x|+y+f(y+f(y)))=3 y+|f(x)|, \forall x, y \in \mathbb{R}$

Lời giải. Dễ thấy $f$ toàn ánh. Giả sử $f(a)=0$ và thay $x=0, y=a$, ta có

$\quad\quad\quad\quad\quad\quad\quad\quad\quad\quad 0=3 a+|f(0)|$

Suy ra $a$ tồn tại duy nhất và $a=-\frac{1}{3}|f(0)| \leq 0$. Lại thay $x=y=a$, ta có $f(0)=3 a \leq 0$. Lại thay $x=-a, y=a$ thì chú ý rằng $|-a|+a=0$, ta có $f(0)=3 a+|f(-a)|$ nên $f(-a)=0$, điều này kéo theo $a=-a$ hay $a=0$ (do tính duy nhất ở trên).

Thay $y=0$ thì $f(|x|)=|f(x)|$ nên $f(x) \geq 0, \forall x \geq 0$. Xét $x>0$ và $y=-\frac{f(x)}{3}$, ta có $f\left(x-\frac{f(x)}{3}+f\left(-\frac{f(x)}{3}+f\left(-\frac{f(x)}{3}\right)\right)\right)=0$ nên

$\quad\quad\quad\quad\quad\quad\quad\quad\quad\quad -\frac{f(x)}{3}+f\left(-\frac{f(x)}{3}+f\left(-\frac{f(x)}{3}\right)\right)=-x$

với mọi $x>0$. Trong đề bài, thay $x=0$ thì $f(y+f(y+f(y)))=3 y$. Thay $y \rightarrow-\frac{f(x)}{3}$ thì $f\left(-\frac{f(x)}{3}+f\left(-\frac{f(x)}{3}+f\left(-\frac{f(x)}{3}\right)\right)\right)=-f(x)$. So sánh hai đẳng thức trên, ta có $f(-x)=-f(x), \forall x>0$ nên $f$ là hàm số lẻ.

Từ tính chất hàm số lẻ, ta có $f\left(\frac{f(x)}{3}+f\left(\frac{f(x)}{3}+f\left(\frac{f(x)}{3}\right)\right)\right)=f(x)$ với mọi $x>0$. Trong đề bài, xét $x \geq 0$ và $y \rightarrow \frac{f(y)}{3}$, ta có

$\quad\quad\quad\quad\quad\quad\quad\quad\quad\quad f\left(x+\frac{f(y)}{3}+f\left(\frac{f(y)}{3}+f\left(\frac{f(y)}{3}\right)\right)\right)=f(y)+f(x)$

hay $f(x+y)=f(x)+f(y)$ với mọi $x, y>0$. Vì $f$ cộng tính trên $\mathbb{R}^{+}$nên ta có $f(x)=a x, \forall x>0$. Lại do tính chất hàm lẻ, ta suy ra $f(x)=a x, \forall x \in \mathbb{R}$. Thay vào đề bài, ta có $a=1$.

Vậy tất cả các hàm số cần tìm là $f(x)=x$.

Bài 3. Cho $p$ là số nguyên tố lẻ và $n=2 p+r$ với $r \in{0,1,2, \ldots, p-1}$. Đặt $X={1,2, \ldots, n}$. Ánh xạ $f: X \rightarrow X$ được gọi là có tính chất $\mathcal{P}$ nếu $f$ không phải là ánh xạ đồng nhất và $f(f(\ldots(f(k)) \ldots)$ ) $=k$ (ánh xạ hợp $p$ lần) với mọi $k \in X$.

Đặt $A_f={k \in X \mid f(k)=k}$.

a) Chứng minh rằng nếu $f$ có tính chất $\mathcal{P}$ thì $\left|A_f\right| \equiv r(\bmod p)$.

b) Gọi $d$ là số các ánh xạ có tính chất $\mathcal{P}$. Chứng minh rằng $d$ không là ước của $n$ !.

(Kí hiệu $|A|$ chỉ số lượng các phần tử của tập hợp $A$.)

Lời giải. a) Ta có

$\quad\quad\quad\quad\quad\quad\quad\quad\quad\quad \left|A_f\right| \equiv r \quad(\bmod p) \Leftrightarrow\left|X \backslash A_f\right| \text { chia hết cho } p \text {. }$

Điều này tương đương số phần tử của tập hợp $B={k \in X \mid f(k) \neq k}$ là bội của $p$. Đặt $f_m(x)$ là ánh xạ hợp $m$ lần. Xét $x \in B$ thì cũng có các số $f(x), f_2(x), \ldots, f_{p-1}(x) \in$ B. Thật vậy,

Giả sử tồn tại $1<m<p$ sao cho $f_m(x)=x$ với số $x \in B$ nào đó, ta chọn $m$ là số nhỏ nhất như thế. Vì $p$ nguyên tố lẻ nên $p$ không chia hết cho $m$. Do vậy tồn tại số $t$ sao cho $0<p-t m<m$. Lại có

$\quad\quad\quad\quad\quad\quad\quad\quad\quad\quad f_m(x)=x \Rightarrow f_{t m}(x)=x \Rightarrow f_{p-t m}(x)=f_p(x)=x$

(mâu thuẫn với tính nhỏ nhất của $m$ ). Vì thế nên với mọi $m$ mà $1<m<p$ thì $f_m(x) \neq x$. Từ đó suy ra với mọi $1<k<l<p$ thì $f_k(x) \neq f_l(x)$, tức là $x, f(x), f_2(x), \ldots, f_{p-1}(x)$ là $p$ số khác nhau thuộc $B$.

Xét số $y \in B$ và $y$ khác tất cả $p$ số ở trên. Khi đó, ta cũng sẽ có $y$ sinh ra một bộ $p$ số phân biệt mới. Giả sử rằng có $f_i(x)=f_j(y)$ với $i<j$ nào đó thì sẽ có $f_{p+i-j}(x)=f_p(y)=y$, mâu thuẫn. Suy ra trong $B$ sẽ có 1 hoặc 2 bộ $p$ số rời nhau, chứng tỏ rằng số phần tử của $B$ chia hết cho $p$. Suy ra điều phải chứng minh.

(b) Từ đây ta thấy rằng để đếm số ánh xạ $f$ có tính chất $\mathcal{P}$, trước hết, ta chọn ra $r$ hoặc $p+r$ vị trí cố định. Ta xét hai trường hợp như sau:

  1. Nếu $\left|A_f\right|=p+r$ thì có $C_n^{p+r}$ cách chọn ra các số này, còn lại $p$ số thì $f$ phải là song ánh trên tập con đó. Do đó trong trường hợp này có $p ! C_n^{p+r}$ cách.
  2. Nếu $\left|A_f\right|=r$ thì tương tự trên, ta cũng đếm được $(p !)^2 C_n^r C_{2 p}^p$.

Từ đó suy ra số ánh xạ tính chất $\mathcal{P}$ là

$\quad\quad\quad\quad\quad\quad\quad\quad\quad\quad d=p ! C_n^{p+r}+(p !)^2 C_n^r C_{2 p}^p$

Ta sẽ chứng minh số này không là ước của $n$ !. Ta viết số $d$ dưới dạng khai triển

$\quad\quad\quad\quad\quad\quad\quad\quad\quad\quad d=p ! \frac{n !}{(p+r) ! p !}+(p !)^2 \frac{n !}{r !(2 p) !} \cdot \frac{(2 p) !}{(p !)^2}=\frac{n !}{(p+r) !}+\frac{n !}{r !} .$

Đặt $(p+r) !=k \cdot(r !)^2$ với $k=\frac{(p+r) !}{(r !)^2}=\frac{p !}{r !} \cdot \frac{(p+r) !}{p ! r !}=\frac{p !}{r !} C_{p+r}^r \in \mathbb{Z}$. Khi đó, ta viết lại

$\quad\quad\quad\quad\quad\quad\quad\quad\quad\quad \frac{n !}{d}=\frac{r !(p+r) !}{r !+(p+r) !}=\frac{k \cdot(r !)^3}{(1+k \cdot r !) \cdot r !}=\frac{k \cdot(r !)^2}{k \cdot r !+1} .$

Dễ thấy số này không thể nguyên vì $k \cdot r !+1$ nguyên tố cùng nhau với $k \cdot(r !)^2$. Từ đó ta có $d$ không là ước của $n$ !.

Nhận xét. Bài này nếu tổng quát $n=k q+r$ thì kết quả câu a vẫn đúng. Tuy nhiên, câu b biến đổi sẽ phức tạp hơn nhiều.

Bài 4. Cho tam giác $A B C$ nội tiếp đường tròn $(O)$ có $A$ cố định và $B, C$ thay đổi trên $(O)$ sao cho $B C$ luôn song song với một đường thẳng cố định. Các tiếp tuyến của $(O)$ tại $B$ và $C$ cắt nhau tại $K$. Gọi $M$ là trung điểm của $B C, N$ là giao điểm của $A M$ với $(O)$. Chứng minh rằng đường thẳng $K N$ luôn đi qua một điểm cố định.

Lời giải. Giả sử $K N$ cắt $(O)$ tại $I$ thì tứ giác $B N C I$ điều hòa.

Do đó $A(B C, N I)=-1$, mà $A N$ chia đôi $B C$ nên $A I | B C$, tức là $A I$ có phương cố định. Từ đó ta thấy $I$ là điểm cố định cần tìm.

Ngày thi thứ hai

Bài 5. Chứng minh rằng nếu $a, b, c$ là độ dài ba cạnh của một tam giác thì:

$\quad\quad\quad\quad\quad\quad\quad\quad\quad\quad (2 a+2 b-c)(2 b+2 c-a)(2 c+2 a-b)>25 a b c .$

Lời giải. Đặt $a+b-c=x, b+c-a=y, c+a-b=z$ thì $x, y, z>0$. Ta đưa về bất đẳng thức

$\quad\quad\quad\quad\quad\quad\quad\quad\quad\quad \left(4 \cdot \frac{x}{y+z}+1\right)\left(4 \cdot \frac{y}{z+x}+1\right)\left(4 \cdot \frac{z}{x+y}+1\right)>25 .$

Không mất tính tổng quát, giả sử $0<x \leq y \leq z$. Đặt $S=x+y+z$. Ta đưa về

$\quad\quad\quad\quad\quad\quad\quad\quad\quad (S+3 x)(S+3 y)(S+3 z)>25(S-x)(S-y)(S-z) .$

Khai triển và rút gọn, ta được

$\quad\quad\quad\quad\quad\quad\quad\quad\quad\quad\quad\quad S^3-4 S(x y+y z+z x)+13 x y z>0 .$

Chú ý rằng

$\quad\quad\quad\quad S^3-4 S(x y+y z+z x)=S\left(S^2-4(x y+y z+z x)\right)=S\left((x+y-z)^2-4 x y\right)$

nên ta đưa về $S(x+y-z)^2+x y(13 z-4 S)>0$. Bất đẳng thức cuối đúng vì $13 c-4 S=9 z-4(x+y)>0$.

Bài 6. Cho dãy số $\left(u_n\right)$ thoả mãn $u_1=\sqrt{2}$ và $u_{n+1}=\frac{2 u_n^2+5 u_n+5}{2 u_n+4}, \forall n \geq 1$. Tìm $\lim \frac{u_n^2-3 u_n+5}{3 n^2+4 n-1}$.

Lời giải. Ta thấy rằng $u_n>0, \forall n$ và $u_{n+1}-u_n=\frac{u_n+5}{2\left(u_n+2\right)}>0$ nên dãy tăng. Giả sử dãy bị chặn trên thì nó hội tụ về $L>0$, suy ra

$\quad\quad\quad\quad\quad\quad\quad\quad\quad L=\frac{2 L^2+5 L+5}{2 L+4} \Leftrightarrow L=-5,$

vô lý. Suy ra $\lim _{n \rightarrow+\infty} u_n=+\infty$. Từ đó, ta được

nên theo định lý Stolz, ta suy ra $\lim _{n \rightarrow+\infty} \frac{u_n}{n}=\frac{1}{2}$ và $\lim _{n \rightarrow+\infty} \frac{u_n}{n^2}=0$. Do đó, trong biểu thức cần tính giới hạn, chia tử và mẫu cho $n^2$ rồi áp dụng kết quả trên, ta có

$\quad\quad\quad\quad\quad \lim _{n \rightarrow+\infty} \frac{u_n^2-3 u_n+5}{3 n^2+4 n-1}=\lim _{n \rightarrow+\infty} \frac{\left(\frac{u_n}{n}\right)^2-\frac{3 u_n-5}{n^2}}{3+\frac{4}{n}-\frac{1}{n^2}}=\left(\frac{1}{2}\right)^2 \cdot \frac{1}{3}=\frac{1}{12}$

Bài 7. Xét số tự nhiên $n>1$. Bắt đầu từ bộ số $1,2, \ldots, 2 n-1,2 n$, ta thực hiện phép biến đổi sau: Chọn hai số $a, b$ sao cho $a-b>1$, xoá hai số này và thay thế bởi hai số $a-1, b+1$. Với bộ số mới, ta lại tiếp tục thực hiện phép biến đổi tương tự.

a) Chứng minh rằng ta sẽ đạt đến trạng thái dừng, tức là không thể tiếp tục thực hiện phép biến đổi như vậy được nữa.

b) Gọi $k$ là số lần phép biến đổi cần thực hiện để đạt đến trạng thái dừng. Tìm giá trị nhỏ nhất và lớn nhất của $k$.

Lời giải. (a) Xét đại lượng $S$ là tổng bình phương các số thu được sau mỗi thao tác biến đổi.

Ta thấy rằng từ $(a, b)$ với $a-b>1$, ta đưa về bộ $(a-1, b+1)$ thì tổng trên thay đổi một lượng là $a^2+b^2-(a-1)^2-(b+1)^2=2(a+b-1)>0$. Do đó, tổng $S$ giảm ngặt, và rõ ràng $S$ phải luôn dương nên thao tác trên chỉ thực hiện được trong hữu hạn lần.

(b) Rõ ràng tổng trên không đổi khi không còn cặp số $a, b$ nào mà $a-b>1$. Điều này đồng nghĩa với việc các số thu được trong trạng thái cuối chỉ nhận hai giá trị liên tiếp nào đó. Ta thấy rằng tổng các số đã cho luôn không đổi và là $1+2+\cdots+2 n=n(2 n+1)$

Giả sử cuối cùng, ta có $x$ số $m$ và $y$ số $m+1$ thì

$\quad\quad\quad\quad\quad\quad\quad\quad\quad\quad\quad\quad\left\{\begin{array}{l}x+y=2 n \\ m x+(m+1) y=n(2 n+1)\end{array}\right.$

Suy ra $2 m n+y=2 n^2+n \Rightarrow n \mid y$. Tuy nhiên, nếu $y \in{0,2 n}$ thì vô lý vì vế phải không chia hết cho $2 n$. Do đó $x=y=n$ và $m=n$, tức là ở trạng thái cuối, ta còn $n$ số $n$ và $n+1$.

  • Tổng bình phương của chúng là $S=n \cdot n^2+n \cdot(n+1)^2=n\left(2 n^2+2 n+1\right)$.
  • Tổng bình phương ban đầu là $S_0=1^2+2^2+\cdots+(2 n)^2=\frac{n(2 n+1)(4 n+1)}{3}$.

Suy ra $S_0-S=\frac{2}{3}\left(n^3-n\right)$.

(b) Để thực hiện được nhiều lần nhất thì giá trị giảm đi ở mỗi lần phải ít nhất. Theo câu a) thì giá trị đó sẽ là $2(a+b-1) \geq 2$.

Suy ra số lần nhiều nhất sẽ là $\frac{1}{3}\left(n^3-n\right)$. Để thực hiện được điều này, ta sẽ cố gắng trong mỗi thao tác tạo ra nhiều giá trị nhất có thể và đồng thời làm giảm số lượng các giá trị ở hai biên đi. Từ đó ta được $k_{\max }=\frac{1}{3}\left(n^3-n\right)$.

Để thực hiện được ít lần nhất, ta sử dụng ý tưởng tham lam, mỗi lần, ta sẽ chọn các cặp số nằm về hai phía của $n, n+1$. Khi đó, giá trị của các số $1,2, \ldots, n-1$ sẽ dần dần được tăng lên, trong khi giá trị của các số $n+2, n+3, \ldots, 2 n$ dần dần sẽ giảm đi. Tổng khoảng cách từ các số nhỏ hơn $n$ đến $n$ là $1+2+\cdots+n-1=\frac{n(n-1)}{2}$. Tương tự thì tổng khoảng cách các số lớn hơn $n+1$ đến $n+1$ cũng là $\frac{n(n-1)}{2}$. Ta thấy mỗi lần thao tác thì các số này sẽ thu hẹp khoảng cách đúng 2 đơn vị nên số lần thao tác tối thiểu phải là $\frac{1}{2}\left(\frac{n(n-1)}{2}+\frac{n(n-1)}{2}\right)=\frac{n(n-1)}{2}$.

Để đạt được giá trị này, mỗi lần, ta chỉ cần chọn các cặp số có dạng $(t, 2 n+1-t)$ với $1 \leq t \leq n-1$ là được. Suy ra $k_{\min }=\frac{n(n-1)}{2}$.

Bài 8. Cho đường tròn $\left(\gamma_1\right)$ đường kính $A B$ và đường tròn $\left(\gamma_2\right)$ tâm $A$ cắt $\left(\gamma_1\right)$ tại $C, D$. Điểm $M$ thay đổi trên cung $C D$ (nằm bên trong $\left(\gamma_1\right)$ ) của $\left(\gamma_2\right)$. Gọi $B M$ cắt $\left(\gamma_1\right)$ tại $N$ khác $M$ và $B$. Tìm giá trị nhỏ nhất của $\frac{N D+N C}{M N}$.

Lời giải. Theo định lý Ptolemy cho tứ giác $B C N D$ nội tiếp trong $\gamma_1$ thì

$\quad\quad\quad\quad\quad\quad\quad\quad\quad\quad\quad B C \cdot N D+B D \cdot N C=B N \cdot C D .$

Vì $A C=A D$ nên $B C=B D=m$ và $C D=n$ là các giá trị cố định.

Ta có

$\quad\quad\quad\quad\quad\quad\quad\quad m(N C+N D)=n \cdot B N \Rightarrow N C+N D=\frac{n}{m} \cdot B N .$

Suy ra $\frac{N C+N D}{M N}=\frac{n}{m} \cdot \frac{B N}{M N}$. Ta đưa về tìm giá trị nhỏ nhất của $\frac{B N}{M N}$. Xét phương tích từ $B$ đến $\gamma_2$ thì $B M \cdot B N=B K \cdot B A=c$ là hằng số nên$(B N-M N) B N=c$. Do đó $\frac{M N}{B N}=1-\frac{c}{B N^2}$ nên

$\quad\quad\quad\quad\quad\quad\frac{B N}{M N} \min \Leftrightarrow \frac{M N}{B N} \max \Leftrightarrow \frac{c}{B N^2} \min \Leftrightarrow B N^2 \max .$

Dễ thấy $\max B N=A B$, xảy ra khi $N \equiv A$ hay $M \equiv K$. Khi đó

$\quad\quad\quad\quad\quad\quad\quad\quad\quad\quad\quad\frac{N C+N D}{M N}=\frac{A C+A D}{A K}=2$

chính là giá trị nhỏ nhất cần tìm.

 

 

 

 

 

 

 

 

 

 

 

 

 

 

 

 

 

 

 

 

 

 

 

 

 

 

 

 

 

 

 

 

 

 

 

 

 

 

 

 

 

 

 

 

 

 

 

 

 

 

 

 

 

 

 

 

 

 

 

 

 

 

 

 

 

 

 

 

 

 

 

 

 

 

 

 

 

 

 

 

 

 

 

 

 

 

 

 

 

 

 

 

 

 

 

 

 

Đề thi và đáp án kì thi chọn đội tuyển thi Quốc gia trường Phổ thông Năng khiếu năm học 2009 – 2010

ĐỀ THI

Ngày thi thứ nhất

Bài 1. Cho $a, b, c$ là các số thực để đa thức $P(x)=x^4+ax^3+b x^2+c x+1$ có ít nhất một nghiệm thực. Tìm tất cả các bộ $(a, b, c)$ để $a^2+b^2+c^2$ đạt giá trị nhỏ nhất.

Bài 2. Cho $A={1,2, \ldots, 2 n}$. Một tập con của $A$ được gọi là tốt nếu như có đúng 2 phần tử $x, y$ và đồng thời $|x-y| \in[1, n]$. Tìm số các tập hợp $[A_1, A_2, \ldots, A_n]$ để $A_i$ là tập con tốt của $A$ với $1 \leq i \leq n$ và $\bigcup_{i=1}^n A_i=A$.

Bài 3. Tìm tất cả các hàm số $f: \mathbb{N}^* \rightarrow \mathbb{N}^*$ thoả mãn các điều kiện sau:

$\quad\quad(i) f $ là hàm số tăng thật sự trên $\mathbb{N}^*$.

$\quad\quad(ii) f(f(n))=4 n+9 \forall n \in \mathbb{N}^*$.

$\quad\quad(iii) f(f(n)-n)=2 n+9 \forall n \in \mathbb{N}^*$.

Bài 4. Cho đường tròn tâm $O$ và dây cung $A B$ cố định khác đường kính. Một điểm $P$ thay đổi trên cung lớn $A B$. Gọi $I$ là trung điểm của $A B$. Lấy các điểm $M, N$ trên các tia $P A, P B$ sao cho $\angle P M I=\angle P N I=\angle A P B$.

(a) Chứng minh rằng đường cao từ $P$ của tam giác $P M N$ luôn đi qua một điểm cố định.

(b) Chứng minh rằng đường thẳng Euler của tam giác $P M N$ luôn đi qua một điểm cố định.

Ngày thi thứ hai

Bài 5. Cho $a, b, c$ là các số thực dương. Giải hệ phương trình sau:

$\quad\quad\quad\quad\quad\quad\quad\quad\quad\quad\left\{\begin{array}{l}a x-a b y+\frac{1}{x y}=b c^2 \\ a b z-b c^2 x+\frac{1}{x z}=a . \\ b c^2 y-a z+\frac{1}{y z}=a b\end{array}\right.$

Bài 6. Cho dãy số $\left(a_n\right)$ xác định bởi $a_1=a, a_{n+1}=\left(a_1+\cdots+a_n-2\right)^2 \forall n \in \mathbb{N}^*$. Đặt $S_n=a_1+a_2+\cdots+a_n$. Tìm tất cả các giá trị $a$ để dãy số $\left(S_n\right)$ hội tụ.

Bài 7. Tìm tất cả các số nguyên dương $k$ để phương trình sau có nghiệm nguyên dương $(x, y)$ :

$$\quad\quad x^2+y^2+x+y=k x y$

Bài 8. Cho tam giác $A B C$ nội tiếp đường tròn $(O)$. Gọi $I, I_1, I_2, I_3$ lần lượt là tâm đường tròn nội tiếp và tâm đường tròn bàng tiếp các đỉnh $A, B, C$ của tam giác $A B C$. Dường tròn ngoại tiếp tam giác $I I_2 I_3$ cắt $(O)$ tại hai điểm $M_1, N_1$. Gọi $J_1$ là giao điểm của $A I$ và $(O)$. Ký hiệu $d_1$ là đường thẳng qua $J_1$ và vuông góc với $M_1 N_1$. Xác định các đường thẳng $d_2, d_3$ tương tự. Chứng minh rằng $d_1, d_2, d_3$ dồng quy.

 

LỜI GIẢI

Ngày thi thứ nhất

Bài 1. Cho $a, b, c$ là các số thực để đa thức $P(x)=x^4+a x+3+b x^2+c x+1$ có ít nhất một nghiệm thực. Tìm tất cả các bộ $(a, b, c)$ để $a^2+b^2+c^2$ đạt giá trị nhỏ nhất.

Lời giải. Gọi $x_0$ là một nghiệm của $P(x)$ (dễ thấy $x_0 \neq 0$ ). Do $P\left(x_0\right)=0$ nên ta có

$\quad\quad\quad\quad\quad\quad\quad\quad\quad\quad-\left(x_0^4+1\right)=a x_0^3+b x_0^2+c x_0 .$

Sử dụng bất đẳng thức Cauchy-Schwarz, ta có

$\quad\quad\quad\quad\quad\left(x_0^4+1\right)^2=\left(a x_0^3+b x_0^2+c x_0\right)^2 \leq\left(a^2+b^2+c^2\right)\left(x_0^6+x_0^4+x_0^2\right) .$

Đặt $t=x_0^2>0$. Từ đánh giá trên, ta suy ra

$\quad\quad\quad\quad\quad\quad\quad\quad\quad\quad a^2+b^2+c^2 \geq \frac{\left(t^2+1\right)^2}{t^3+t^2+t}=\frac{\left(t^2+1\right)^2}{t\left(t^2+t+1\right)}$

Mà theo bất đẳng thức AM-GM thì

$\quad\quad\quad\quad\quad\quad\quad\quad\quad\quad t \leq \frac{t^2+1}{2} \text { và } t^2+t+1 \leq t^2+\frac{t^2+1}{2}=\frac{3}{2}\left(t^2+1\right) .$

Do đó

$\quad\quad\quad\quad\quad\quad\quad\quad\quad\quad\frac{\left(t^2+1\right)^2}{t\left(t^2+t+1\right)} \geq \frac{4}{3}, \text { nên } a^2+b^2+c^2 \geq \frac{4}{3}$

Đẳng thức xảy ra khi và chỉ khi

$\quad\quad\quad\quad\quad\quad\quad\quad\quad\quad\left\{\begin{array}{l}x_0^4+a x_0^3+b x_0^2+c x_0+1=0 \\ x_0^2=1 \\ \frac{a}{x_0^3}=\frac{b}{x_0^2}=\frac{c}{x_0}\end{array}\right.$

Giải hệ này, ta thu được $a=b=c=-\frac{2}{3}$ hoặc $a=-b=c=\frac{2}{3}$.

Bài 2. Cho $A=[1,2, \ldots, 2 n]$. Một tập con của $A$ được gọi là tốt nếu như có đúng 2 phần tử $x, y$ và đồng thời $|x-y| \in[1, n]$. Tìm số các tập hợp $[A_1, A_2, \ldots, A_n]$ để $A_i$ là tập con tốt của $A$ với $1 \leq i \leq n$ và $\bigcup_{i=1}^n A_i=A$.

Lời giải . Gọi $u_n, n \in{1,2, \ldots, n}$ là số các tập hợp $[A_1, A_2, \ldots, A_n]$ thỏa mãn yêu cầu đề bài, đồng thời hai phần tử $n$ và $n+1$ không đi cùng nhau trong bất kì tập $A_i$ nào. Ta chia các số $1,2, \ldots, 2 n$ vào một bảng $2 \times n$ như sau

$\quad\quad\quad\quad\quad\quad\quad\quad\quad\quad\quad\quad\quad\quad\begin{array}{|c|c|c|c|}\hline 1 & 2 & \ldots & n \\ \hline n+1 & n+2 & \ldots & 2 n \\ \hline\end{array}$

Khi đó, mỗi cách chọn được liệt kê trong $u_n$ tương ứng với một cách chọn từ bảng trên các cặp gồm hai số ở cùng một cột hoặc hai số liên tiếp nhau trên cùng một hàng. Xét $u_{n+1}$, vì phần tử $2(n+1)$ chỉ có thể đi cùng với $n+1$ hoặc $2 n+1$ trong cùng một tập $A_i$ nào đó nên ta xét hai khả năng sau.

  • $2(n+1)$ và $n+1$ cùng thuộc một tập $A_i$, giả sử là $A_{n+1}$.

$\quad\quad\quad\quad\quad\quad\quad\quad\quad\quad\quad\quad\begin{array}{|c|c|l|c|}\hline 1 & 2 & \ldots & n+1 \\ \hline n+1 & n+2 & \ldots & 2(n+1) \\ \hline\end{array}$

Lúc này, mỗi cách chọn một bộ $[A_1, A_2, \ldots, A_n]$ ứng với một cách chọn các cặp số gồm các số ở cùng một cột hoặc ở cạnh nhau trong cùng một hàng từ một bảng $2 \times n$. Theo định nghĩa của ta số cách chọn như thế là $u_n$. Vậy trong trường hợp này có $u_n$ cách chọn.

  • $2(n+1)$ và $2 n+1$ cùng thuộc một tập $A_i$, giả sử đó là $A_{n+1}$.

$\quad\quad\quad\quad\quad\quad\quad\quad\quad\quad\quad\quad\begin{array}{|c|c|l|c|c|}\hline 1 & 2 & \ldots & n & n+1 \\ \hline n+1 & n+2 & \ldots & 2 n+1 & 2(n+1) \\ \hline\end{array}$

Ta thấy $n+1$ chỉ có thể đi cùng với $2(n+1)$ (trường hợp $n+1$ đi cùng với $n+2$ không được xét trong $\left.u_{n+1}\right)$ nhưng $2(n+1)$ đã đi cùng với $2 n+1$ nên $n+1$ phải đi cùng với $n$ trong cùng một tập $A_i$ nào đó, giả sử là $A_n$. Lập luận tương tự trường hợp trên, ta suy ra số cách chọn các tập ${A_1, A_2, \ldots, A_n-1}$ là $u_{n-1}$.

Theo quy tắc cộng, ta có $u_{n+1}=u_n+u_{n-1}$. Mặt khác, $u_1=1$ và $u_2=2$ nên ta tìm được công thức tổng quát của $u_n$ là

$\quad\quad\quad\quad\quad\quad\quad\quad\quad\quad\quad\quad u_n=\frac{1}{\sqrt{5}}\left[\left(\frac{1+\sqrt{5}}{2}\right)^{n+1}-\left(\frac{1-\sqrt{5}}{2}\right)^{n+1}\right]$

Xét trường hợp sinh ra bộ $[A_1, A_2, \ldots, A_n]$ có $n$ và $n+1$ đi cùng trong một tập $A_i$ nào đó.

$\quad\quad\quad\quad\quad\quad\quad\quad\quad\quad\quad\quad\begin{array}{|c|c|l|c|}\hline 1 & 2 & \ldots & n \\ \hline n+1 & n+2 & \ldots & 2 n \\ \hline\end{array}$

Rõ ràng 1 chỉ có thể đi cùng với 2 hoặc $n+1$ nhưng $n+1$ đã đi cùng $n$ nên 1 chỉ có thể đi cùng với 2 . Tiếp theo, $n+2$ có thể đi cùng với $2, n+1$ hay $n+3$ nhưng 2 đã đi với 1 còn $n+1$ đã đi với $n$ nên $n+2$ phải đi với $n+3$.

Tiếp tục, 3 có thể đi cùng 2,4 hay $n+3$ nhưng 2 đã đi với 1 còn $n+3$ đã đi với $n+2$ nên 3 phải đi với 4 . Tiếp tục lý luận như trên, ta suy ra $A_i$ phải có dạng

$\quad\quad\quad [1,2],[3,4], \ldots,[n-2, n-1],[n, n+1],[n+1, n+2], \ldots,[2 n-1,2 n]$

Từ đó suy ra trường hợp này chỉ cho ta duy nhất một bộ $[A_1, A_2, \ldots A_n]$ nếu $n$ lẻ và không có bộ nào nếu $n$ chẵn.

Vậy số các bộ $[A_1, A_2, \ldots A_n]$ thỏa mãn đề bài là

$\quad\quad\quad\quad\quad\quad\quad\quad\begin{cases}\frac{1}{\sqrt{5}}\left[\left(\frac{1+\sqrt{5}}{2}\right)^{n+1}-\left(\frac{1-\sqrt{5}}{2}\right)^{n+1}\right] & , n=1,n \text { chẳn. } \\ 1+\frac{1}{\sqrt{5}}\left[\left(\frac{1+\sqrt{5}}{2}\right)^{n+1}-\left(\frac{1-\sqrt{5}}{2}\right)^{n+1}\right] & , n>1, n \text { lẻ’ }\end{cases}$

Bài 3. Tìm tất cả các hàm số $f: \mathbb{N}^* \rightarrow \mathbb{N}^*$ thoả mãn các điều kiện sau:

$\quad\quad(i) f $ là hàm số tăng thật sự trên $\mathbb{N}^*$.

$\quad\quad(ii) f(f(n))=4 n+9 \forall n \in \mathbb{N}^*$.

$\quad\quad(iii) f(f(n)-n)=2 n+9 \forall n \in \mathbb{N}^*$.

Lời giải. Vì $f: \mathbb{N}^* \longrightarrow \mathbb{N}^*$ tăng ngặt nên

$\quad\quad\quad\quad\quad\quad\quad\quad\quad\quad f(a)-f(b) \geq a-b, \forall a, b \in \mathbb{N}^*, a>b .$

Theo điều kiện (iii), ta có

$2=2(n+1)+9-(2 n+9) =f(f(n+1)-(n+1))-f(f(n)-n) $

$\quad\quad\quad\quad\quad\quad\quad\quad\quad\quad\quad\quad \geq f(n+1)-(n+1)-[f(n)-n] $

$\quad\quad\quad\quad\quad\quad\quad\quad\quad\quad\quad\quad =f(n+1)-f(n)-1 .$

Do đó $f(n+1)-f(n) \leq 3$

với mọi $n \in \mathbb{N}^*$, tức $f(n+1)-f(n) \in\ {1,2,3}$

với mọi $n \in \mathbb{N}^*$. Ta xét các trường hợp sau

  • Giả sử tồn tại $n \in \mathbb{N}^*$ sao cho $f(n+1)-f(n)=1$ thì

$\quad\quad\quad\quad\quad\quad\quad\quad\quad\quad f(n+1)-(n+1)=f(n)-n,$

suy ra $2(n+1)+9=f(f(n+1)-(n+1))=f(f(n)-n))=2 n+9$, vô lí.

  • Giả sử tồn tại $n \in \mathbb{N}^*$ sao cho $f(n+1)-f(n)=3$ thì

$\quad\quad\quad\quad\quad\quad\quad\quad\quad\quad f(n+1)-(n+1)=f(n)-n+2$

Ta lại có

$\quad\quad\quad\quad\quad\quad\quad\quad\quad\quad f(f(n+1)-(n+1))-f(f(n)-n)=2$

Do đó, nếu $f(n)>n$ thì đặt $t=f(n)-n \in \mathbb{N}^*$, ta suy ra $f(t+2)=t+2$.

Mà $f$ tăng ngặt nên $f(t+1)-f(t)=1$, mâu thuẫn.

Vậy $f(n) \leq n$ với mọi $n \in \mathbb{N}^*$ suy ra

$f(n)=n$ với mọi $n \in \mathbb{N}^*$.

Khi đó ta lại có $n=f(n)=f(f(n))=4 n+9,$ tức $n=-3$, vô lí.

Như vậy, $f(n+1)-f(n) \notin{1,3}$ với mọi $n \in \mathbb{N}^*$, nên

$\quad\quad\quad\quad\quad\quad\quad\quad\quad\quad f(n+1)-f(n)=2 \text {, với mọi } n \in \mathbb{N}^* \text {. }$

Ta suy ra $f(n)=2 n+k$. Thay vào đề bài, ta thu được $k=3$. Vậy $f(n)=2 n+3$ là nghiệm duy nhất của phương trình.

Bài 4. Cho đường tròn tâm $O$ và dây cung $A B$ cố định khác đường kính. Một điểm $P$ thay đổi trên cung lớn $A B$. Gọi $I$ là trung điểm của $A B$. Lấy các điểm $M, N$ trên các tia $P A, P B$ sao cho $\angle P M I=\angle P N I=\angle A P B$.

(a) Chứng minh rằng đường cao từ $P$ của tam giác $P M N$ luôn đi qua một điểm cố định.

(b) Chứng minh rằng đường thẳng Euler của tam giác $P M N$ luôn đi qua một điểm cố định.

Lời giải. (a) Kí hiệu $X=M I \cap P B, Y=N I \cap P A$. Ta có $\angle P M I=\angle P N I=$ $\angle A P B$ nên các tam giác $P M X$ và $P N Y$ cân tại $X, Y$. Từ đó suy ra

$\angle P X M=\angle P Y N=180^{\circ}-2 \angle A P B,$

suy ra $M, N, X, Y$ đồng viên. Gọi $S$ là tâm đường tròn ngoại tiếp tam giác $A O B$ thì $S$ cố định. Ta có $\angle I S B=180^{\circ}-\angle A O B=180^{\circ}-2 \angle A P B=\angle P X M$. Tương tự, ta suy ra $\angle I S A=\angle P Y N$. Do đó $I, S, X, B$ đồng viên và $I, S, Y, A$ đồng viên. Suy ra

$\angle S X B=\angle S Y A=\angle S I B=90^{\circ} .$

Suy ra $I S$ là đường kính của đường tròn ngoại tiếp tam giác $P X Y$. Mặt khác, $M, N, X, Y$ dồng viên nên nên $M N$ và $X Y$ đối song nhau trong $\angle A P B$, tức $I S \perp$ $M N$. Nói cách khác, đường cao từ $P$ của tam giác $P M N$ đi qua điểm $S$ cố định.

(b) Trước tiên, ta chứng minh bổ đề sau.

BỔ ĐỀ. Cho tam giác $A B C$ và đường tròn $(\omega)$ đi qua hai điểm $B, C$ và cắt các canh $A B, A C$ tại $X, Y$. Gọi $X X^{\prime}, Y Y^{\prime}$ là các đường cao của tam giác $A X Y$. Gọi $B B^{\prime}, C C^{\prime}$ là các đường cao của tam giác $A B C$. Gọi $H, H^{\prime}$ là các trục tâm của tam giác $A B C$ và tam giác $A X Y$. Kí hiệu $I \equiv B Y \cap C X$. Khi đó $H, I, H^{\prime}$ thẳng hàng.

Chứng minh. Ta có $X, Y, X^{\prime}, Y^{\prime}$ đồng viên nên $\overline{H^{\prime} X} \cdot \overline{H^{\prime} X^{\prime}}=\overline{H^{\prime} Y} \cdot \overline{H^{\prime} Y^{\prime}}$, tức là

$P_{H^{\prime} /[B Y]}=P_{H^{\prime} /[C X]},$

trong đó $[U V]$ là đường tròn đường kính $U V$. Ta có $B, C, B^{\prime}, C^{\prime}$ đồng viên nên $\overline{H B} \cdot \overline{H B^{\prime}}=\overline{H C} \cdot \overline{H C^{\prime}}$, tức

$P_{H /[B Y]}=P_{H /[C X]} .$

Cuối cùng $B, C, X, Y$ đồng viên nên $\overline{I B} \cdot \overline{I Y}=\overline{I C} \cdot \overline{I X}$, tức

$P_{I /[B Y]}=P_{I /[C Y]} .$

Suy ra $H, I, H^{\prime}$ thẳng hàng vì cùng thuộc trục đẳng phương của $[B Y]$ và $[C X]$.

Trở lại bài toán,

Gọi $H, O^{\prime}$ lần lượt là trực tâm và tâm đường tròn ngoại tiếp của tam giác $P M N$. Ta có $O^{\prime} P=O^{\prime} M$ và $X P=X M$ nên $X O^{\prime}$ là đường trung trực của $P M$, suy ra $X O^{\prime} \perp P Y$. Tương tự ta cũng có $Y O^{\prime} \perp P X$.

Vì thế nên $O^{\prime}$ cũng chính là trực tâm của tam giác $P X Y$. Áp dụng bổ đề cho tam giác $P X Y$ với $(\omega) \equiv(M N X Y)$ thì ta có $O^{\prime}, H, I \equiv Y N \cap M X$ thẳng hàng. Hay nói cách khác, đường thẳng Euler $O^{\prime} H$ của tam giác $P M N$ đi qua điểm $I$ cố định. Bài toán được giải quyết.

Ngày thi thứ hai

Bài 5. Cho $a, b, c$ là các số thực dương. Giải hệ phương trình sau:

$\quad\quad\quad\quad\quad\quad\quad\quad\quad\quad\left\{\begin{array}{l}a x-a b y+\frac{1}{x y}=b c^2 \\ a b z-b c^2 x+\frac{1}{x z}=a \\ b c^2 y-a z+\frac{1}{y z}=a b\end{array}\right.$

Lời giải. Đặt $(m, n, p)=\left(a, a b, b c^2\right)$. Khi đó $m, n, p>0$. Hệ phương trình trở thành

$\quad\quad\quad\quad\quad\quad\quad\quad\quad\quad\left\{\begin{array}{l}m x-n y+\frac{1}{x y}=p \\ n z-p x+\frac{1}{z x}=m \\ p y-m z+\frac{1}{y z}=n,\end{array}\right.$

tương đương

$\quad\quad\quad\quad\quad\quad\quad\quad\quad\quad\left\{\begin{aligned}m x-n y-p &=-\frac{1}{x y}, \\ -m+n z-p x &=-\frac{1}{z x}, \\ -m z+p y-n &=-\frac{1}{y z} .\end{aligned}\right.$

Xem hệ trên là hệ phương trình tuyến tính theo ẩn $m, n, p$, ta có

$\quad\quad D=\left|\begin{array}{ccc}x & -y & -1 \\ -1 & z & -x \\ -z & -1 & y\end{array}\right|=x y z-x y z-1-x^2-y^2-z^2=-1-\left(x^2+y^2+z^2\right) \neq 0$

$\quad\quad\quad\quad\quad\quad\quad \quad\quad D_m=\left|\begin{array}{ccc}x & -\frac{1}{x y} & -1 \\ -1 & -\frac{1}{z x} & -x \\ -z & -\frac{1}{y z} & y\end{array}\right|=-\frac{1+x^2+y^2+z^2}{z x} .$

Tương tự, ta cũng tính được

$\quad\quad\quad\quad\quad\quad D_n=-\frac{1+x^2+y^2+z^2}{y z} \text {, và } D_p=-\frac{1+x^2+y^2+z^2}{x y} .$

Do đó

$\quad\quad\quad\quad\quad\quad\quad (m, n, p)=\left(\frac{D_m}{D}, \frac{D_n}{D}, \frac{D_p}{D}\right)=\left(\frac{1}{z x}, \frac{1}{y z}, \frac{1}{x y}\right) .$

Thay $(m, n, p)=\left(a, a b, b c^2\right)$, ta được

$\quad\quad\quad\quad\quad\quad\quad\quad\quad\quad x y=\frac{1}{b c^2}, y z=\frac{1}{a b}, z x=\frac{1}{a} .$

Nhân ba phương trình trên vế theo vế rồi lấy căn bậc hai, ta được $x y z=\pm \frac{1}{a b c}$.

  • Với $x y z=\frac{1}{a b c}$, ta suy ra $x=\frac{1}{c}, y=\frac{1}{b c}, z=\frac{c}{a}$.
  • Với $x y z=-\frac{1}{a b c}$, ta suy ra $x=-\frac{1}{c}, y=-\frac{1}{b c}, z=-\frac{c}{a}$.

Vậy hệ có 2 nghiệm là $\left(\frac{1}{c}, \frac{1}{b c}, \frac{c}{a}\right)$ và $\left(-\frac{1}{c},-\frac{1}{b c},-\frac{c}{a}\right)$.

Bài 6. Cho dãy số $\left(a_n\right)$ xác định bởi $a_1=a, a_{n+1}=\left(a_1+\cdots+a_n-2\right)^2 \forall n \in \mathbb{N}^*$.

Đặt $S_n=a_1+a_2+\cdots+a_n$. Tìm tất cả các giá trị $a$ để dãy số $\left(S_n\right)$ hội tụ.

Lời giải. Từ giả thiết suy ra $S_{n+1}-S_n=\left(S_n-2\right)^2$. Do đó dãy $\left(S_n\right)$ được xác định như sau

$\quad\quad\quad\quad\quad\quad\quad\quad\quad\quad\left\{\begin{array}{l}S_1=a, \\ S_{n+1}=f\left(S_n\right)=S_n^2-3 S_n+4 .\end{array}\right.$

Hơn nữa $f^{\prime}(x)=0$ có nghiệm duy nhất $x=\frac{2}{3}$ nên ta có thể vẽ bảng biến thiên, khảo sát được hàm số này. Từ đó, nhờ việc các trường hợp của $a$, ta thấy

  • Nếu $a>2$. Giả sử $[S_n]$ có giới hạn $L$ thì ta phải có $L=f(L)$ nên $L \in{1,2}$. Mà $\left(S_n\right)$ không giảm nên $L \geq a>2$, mâu thuẫn. Vậy nếu $a>2$ thì $\left(S_n\right)$ không hội tụ.
  • Nếu $a<1$ thì suy ra $S_2=f(a)>2$. Quay về trường hợp 1 , ta suy ra $\left(S_n\right)$ không hội tụ.
  • Nếu $1 \leq a \leq 2$ thì từ bảng biến thiên, ta có $\frac{7}{4} \leq S_n \leq 2$ với mọi $n \in \mathbb{N}^*$. Từ đó $\left(S_n\right)$ không giảm và bị chặn nên $\left(S_n\right)$ hội tụ.

Vậy các giá trị của $a$ thỏa mãn đề bài là $a \in[1,2]$.

Bài 7. Tìm tất cả các số nguyên dương $k$ để phương trình sau có nghiệm nguyên dương $(x, y)$ :

$\quad\quad\quad\quad\quad\quad\quad\quad\quad\quad\quad x^2+y^2+x+y=k x y .$

Lời giải. Không mất tính tổng quát, giả sử $x \geq y$. Xét giá trị $k$ sao cho phương trình đã cho có nghiệm nguyên dương. Trong các nghiệm ấy, gọi $\left(x_0, y_0\right)$ là nghiệm sao cho $x_0 \geq y_0 \geq 0$ và $x_0$ nhỏ nhất. Xét tam thức

$\quad\quad\quad\quad\quad\quad\quad\quad\quad f(x)=x^2-\left(k y_0-1\right) x+y_0^2+y_0 .$

Khi đó $f\left(x_0\right)=0$. Theo định lí Viette, $f(x)$ còn một nghiệm khác là $x_0^{\prime}=k y_0-1-x_0$. Tuy nhiên, theo cách chọn $\left(x_0, y_0\right)$ thì ta có $x_0^{\prime} \geq x_0 \geq y_0$ nên $y_0$ nằm ngoài hai khoảng nghiệm của tam thức bậc hai $f(x)$. Mà hệ số cao nhất của $f(x)$ dương nên $f\left(y_0\right) \geq 0$. Do $f\left(y_0\right) \geq 2 y_0^2+2 y_0-k y_0^2$ nên ta có

$\quad\quad\quad\quad\quad\quad\quad\quad\quad\quad\quad k \leq 2+\frac{2}{y_0} \leq 4 .$

Suy ra $k \in{1,2,3,4}$.

  • Nếu $k=1$ thì phương trình có dạng $x^2+y^2+x+y=x y$, tương đương với

$\quad\quad\quad\quad\quad\quad\quad\quad\left(x-\frac{y}{2}\right)^2+\frac{3}{4} y^2+x+y=0$ (vô lí vì $\left.x, y>0\right)$.

  • Nếu $k=2$ thì phương trình có dạng $x^2+y^2+x+y=2 x y$, tương đương với

$\quad\quad\quad\quad\quad\quad\quad\quad\quad\quad(x-y)^2+x+y=0$ (vô lí vì $x, y>0$ ).

  • Nếu $k=3$ thì phương trình có nghiệm $(x, y)=(2,2)$.
  • Nếu $k=4$ thì phương trình có nghiệm $(x, y)=(1,1)$.

Vậy các giá trị cần tìm là $k=3, k=4$.

Bài 8. Cho tam giác $A B C$ nội tiếp đường tròn $(O)$. Gọi $I, I_1, I_2, I_3$ lần lượt là tâm đường tròn nội tiếp và tâm đường tròn bàng tiếp các đỉnh $A, B, C$ của tam giác $A B C$. Đường tròn ngoại tiếp tam giác $I_2 I_3$ cắt $(O)$ tại hai điểm $M_1, N_1$. Gọi $J_1$ là giao điểm của $A I$ và $(O)$. Ký hiệu $d_1$ là đường thẳng qua $J_1$ và vuông góc với $M_1 N_1$. Xác định các đường thẳng $d_2, d_3$ tương tự. Chứng minh rằng $d_1, d_2, d_3$ đồng quy.

Lời giải. Gọi $\left(O^{\prime}\right)$ là đường tròn ngoại tiếp tam giác $I_1 I_2 I_3$ và $\left(O_1\right)$ là đường tròn ngoại tiếp tam giác $\left(I I_2 I_3\right)$. Ta có $A I \perp I_2 I_3, B I \perp I_3 I_1$ nên $I$ là trực tâm tam giác $I_1 I_2 I_3$ và $(O)$ là đường tròn Euler của tam giác $I_1 I_2 I_3$ nên $O$ là trung điểm của $I O^{\prime}$. Mặt khác thì

$\angle I_2 O_1 I_3=2\left(180^{\circ}-\angle I_2 I I_3\right)=2 \angle I_2 I_1 I_3=I_2 O^{\prime} I_3 .$

Do đó $O^{\prime}$ đối xứng với $O_1$ qua $I_2 I_3$, suy ra $\overrightarrow{O^{\prime} O_1}=\overrightarrow{I_1 I}$, tức $A I O_1 O^{\prime}$ là hình bình hành. Mà $O$ là trung điểm $I O^{\prime}$ nên $O$ cũng là trung điểm của $I_1 O_1$. Hơn nữa, $O O_1 \perp M_1 N_1$ (đường nối tâm vuông góc với dây cung) nên $I_1 O \perp M_1 N_1$.

Mặt khác, $J_1$ là trung điểm của $I I_1$ (đường tròn Euler đi qua trung điểm của đoạn thẳng nối trực tâm của tam giác với đỉnh của tam giác ấy) nên phép vị tự tâm $I$, tỉ số $k=\frac{1}{2}$ biến $I_1 O_1$ thành $d_1$. Do đó $d_1$ đi qua trung điểm $S$ của $O I$. Tương tự, ta suy ra $d_2, d_3$ cũng đi qua $S$, tức $d_1, d_2, d_3$ đồng quy.

Nhận xét. Bài toán thực chất là việc đổi mô hình từ một tính chất quen thuộc liên quan đến trực tâm, chân đường cao sang mô hình ba tâm bàng tiếp. Vì thế, đôi khi việc chuyển đổi giữa các mô hình giúp cho bài toán sáng sủa, dễ xử lý hơn.

 

 

 

 

 

 

 

 

 

 

 

 

 

 

 

 

 

 

 

 

 

 

 

 

 

 

 

 

 

 

 

 

 

 

 

 

 

 

 

 

 

 

 

 

 

 

 

 

 

 

 

 

 

 

 

 

 

 

 

 

 

 

 

 

 

 

 

 

 

 

 

 

 

 

 

 

 

 

 

 

 

 

Đề thi và đáp án kì thi chọn đội tuyển thi Quốc gia trường Phổ thông Năng khiếu năm học 2008 – 2009

ĐỀ THI

Ngày thi thứ nhất

Bài 1.

(a) Chứng minh rằng tồn tại số $n$ chẵn, $n>2008$ sao cho $2009 n-49$ là số chính phương.

(b) Chứng minh rằng không tồn tại số nguyên $m$ sao cho $2009 m-147$ là số chính phương.

Bài 2.

(a) Tìm số các số tự nhiên có 3 chữ số khác nhau và chia hết cho 6 .

(b) Tìm số các số tự nhiên có $n$ chữ số lập từ các chữ số ${1,2,3,4,5}$ và chia hết cho 3 .

Bài 3. Cho tam giác $A B C$ có $A$ cố định và $B, C$ thay đổi trên đường thẳng $d$ cố định sao cho nếu gọi $A^{\prime}$ là hình chiếu của $A$ lên $d$ thì $\overline{A^{\prime} B} \cdot \overline{A^{\prime} C}<0$ và không đổi. Gọi $M$ là hình chiếu của $A^{\prime}$ lên $A C$.

(a) Chứng minh rằng tâm $I$ đường tròn ngoại tiếp tam giác $B C M$ thuộc một đường thẳng cố định.

(b) Gọi $N$ là hình chiếu của $A^{\prime}$ lên $A B$ và $K$ là giao điểm hai tiếp tuyến của $\left(A^{\prime} M N\right)$ tại $M, N$. Chứng minh rằng $K$ thuộc một đường thẳng cố định.

Bài 4. Cho $f(x)=x^2+a x+b$ là tam thức bậc hai với $a, b \in \mathbb{R}$ và $f(f(x))=0$ có 4 nghiệm thực phân biệt. Biết rằng tổng của 2 nghiệm nào đó trong số 4 nghiệm đã nêu bằng $-1$. Chứng minh rằng $b \leq-\frac{1}{4}$.

Ngày thi thứ hai

Bài 5. Giả sử đa thức $P(x)=(x+1)^p(x-3)^q=x^n+a_1 x^{n-1}+a_2 x^{n-2}+\cdots+a_n$, trong đó $p, q$ là các số nguyên dương. Chứng minh rằng nếu như $a_1=a_2$ thì $3 n$ là một số chính phương.

Bài 6.

(a) Cho $a, b, c$ là các số thực dương. Chứng minh rằng ta có bất đẳng thức

$\quad\quad\quad\quad\quad\quad\quad\quad\quad\quad\quad\frac{a^2+b^2+c^2}{a b+b c+c a}+\frac{8 a b c}{(a+b)(b+c)(c+a)} \geq 2 .$

(b) Chứng minh rằng tồn tại các số thực dương $a, b, c$ sao cho

$\quad\quad\quad\quad\quad\quad\quad\quad\quad\quad\quad\frac{a b+b c+c a}{a^2+b^2+c^2}+\frac{(a+b)(b+c)(c+a)}{8 a b c}<2 .$

Bài 7. Cho góc $O x y$ và một điểm $P$ nằm bên trong nó. Gọi $\gamma$ là đường tròn thay đổi nhưng luôn qua $O$ và $P$. Giả sử $\gamma$ cắt $O x, O y$ tại $M, N$. Tìm quỹ tích trọng tâm $G$ và trực tâm $H$ của tam giác $O M N$.

Bài 8. Với mỗi số nguyên dương gọi là tổng các chữ số của $n$.

(a) Chứng minh rằng $n=999, n=2999$ không thể biểu diễn thành tổng $a+b$ mà $S(a)=S(b)$.

(b) Chứng minh rằng với mọi $n$ mà $999<n<2999$ thì điều kiện trên được thỏa mãn.

LỜI GIẢI

Ngày thi thứ nhất

Bài 1.

(a) Chứng minh rằng tồn tại số $n$ chẵn, $n>2008$ sao cho $2009 n-49$ là số chính phương.

(b) Chứng minh rằng không tồn tại số nguyên $m$ sao cho $2009 m-147$ là số chính phương.

Lời giải. Chú ý rằng $2009=49 \cdot 41=7^2 \cdot 41$ nên yêu cầu bài toán tương đương với việc chứng minh

  1. Tồn tại số $n$ chẵn, $n>2008$ sao cho $41 n-1$ là số chính phương.
  2. Không tồn tại số nguyên $m$ sao cho $41 m-3$ là số chính phương.

(a) Trước hết, ta đi tìm một số $a$ sao cho $a^2+1$ chia hết cho 41 . Điều này có thể được thực hiện bằng cách thử tuần tự. Ta dễ dàng tìm được $a=9$ thỏa mãn. Từ đây, ta thấy các số $(82 k+9)^2+1$ là số chẵn và chia hết cho 41 . Bây giờ chỉ cần chọn.

$\quad\quad\quad\quad\quad\quad\quad\quad\quad\quad\quad n=\frac{(82 k+9)^2+1}{41}$

với $k$ đủ lớn là ta tìm được số $n$ thỏa mãn điều kiện đề bài.

(b) Giả sử tồn tại $m$ sao cho $41 m-3=a^2$. Khi đó ta có $-3 \equiv a^2(\bmod 41)$. Từ đó theo định lý Fermat nhỏ thì

$\quad\quad\quad\quad\quad\quad\quad\quad\quad\quad\quad (-3)^{20} \equiv a^{40} \equiv 1 \quad(\bmod 41) .$

Nhưng mặt khác, ta lại có $(-3)^4 \equiv-1(\bmod 41)$, suy ra

$\quad\quad\quad\quad\quad\quad\quad\quad\quad\quad\quad (-3)^{20} \equiv(-1)^5 \equiv-1 \quad(\bmod 41) .$

Do đó $1 \equiv-1(\bmod 41)$, vô lý. Do đó điều đã giả sử là sai, tức là không tồn tại số nguyên $m$ sao cho $41 m-3$ là số chính phương.

Bài 2.

(a) Tìm số các số tự nhiên có 3 chữ số khác nhau và chia hết cho 6 .

(b) Tìm số các số tự nhiên có $n$ chữ số lập từ các chữ số ${1,2,3,4,5}$ và chia hết cho 3 .

Lời giải. (a) Ta sẽ tìm các số có dạng $\overline{a b c}$ sao cho $c$ chẵn và $3 \mid a+b+c$. Ta có các trường hợp sau

  • Nếu $c=0$ thì $a+b \in{3,6,9,12,15,18}$ nên có $2+4+8+6+4+0=24$.
  • Nếu $c=2$ thì $a+b \in{1,4,7,10,13,16}$ nên có $1+3+5+6+6+2=23$.
  • Nếu $c=4$ thì $a+b \in{2,5,8,11,14,17}$ nên có $1+3+7+6+4+2=23$.
  • Nếu $c=6$ thì $a+b \in{3,6,9,12,15,18}$ nên có $3+4+7+6+2+0=22$.
  • Nếu $c=8$ thì $a+b \in{1,4,7,10,13,16}$ nên có $1+3+7+6+4+2=23$.

Do đó, có tất cả $24+23+23+22+23=115$ số thỏa mãn đề bài.

(b) Ta xét khai triển tương ứng

$\quad\quad\quad\quad\quad\quad\quad\quad\quad\quad\quad P(x)=\left(x+x^2+x^3+x^4+x^5\right)^n=\sum_{k=n}^{5 n} a_k x^k$

Số các số chia hết cho ba cần tìm chính bằng tổng các số hạng của khai triển trên có dạng $x^{3 k}$, giả sử tổng đó là $A$. Xét ba nghiệm phức của phương trình $t^3=1$ là $t=1, t=\varepsilon, t=\varepsilon^2$. Ta có:

$\quad\quad\quad\quad\quad\quad\quad\quad\quad\quad\quad P(1)=5^n, P(\varepsilon)=\left(\varepsilon+\varepsilon^2+\varepsilon^3+\varepsilon+\varepsilon^2\right)^n =(-1)^n $

$\quad\quad\quad\quad\quad\quad\quad\quad\quad\quad\quad P\left(\varepsilon^2\right)=\left(\varepsilon^2+\varepsilon+1+\varepsilon^2+\varepsilon\right)^n =(-1)^n$

Hơn nữa, dễ dàng thấy rằng

$\quad\quad\quad\quad\quad\quad\quad\quad\quad\quad\quad P(1)+P(\varepsilon)+P\left(\varepsilon^2\right)=\sum_{k=n}^{5 n} a_k\left(1+\varepsilon^k+\varepsilon^{2 k}\right) $

Nếu $3 \mid k$ thì $P(1)+P(\varepsilon)+P\left(\varepsilon^2\right)=3 A$. Nếu $k$ không chia hết cho 3 thì $1+\varepsilon^k+\varepsilon^{2 k}=$ $1+\varepsilon+e^2=0$ nên các biểu thức còn lại trong tổng $P(1)+P(\varepsilon)+P\left(\varepsilon^2\right)$ đều bằng 0. Vậy nên ta tính được

$\quad\quad\quad\quad\quad\quad\quad\quad\quad\quad\quad P(1)+P(\varepsilon)+P\left(\varepsilon^2\right)=3 A \Leftrightarrow A=\frac{5^n+2(-1)^n}{3}$

Đó cũng chính là số các số cần tìm.

Bài 3. Cho tam giác $A B C$ có $A$ cố định và $B, C$ thay đổi trên đường thẳng $d$ cố định sao cho nếu gọi $A^{\prime}$ là hình chiếu của $A$ lên $d$ thì $\overline{A^{\prime} B} \cdot \overline{A^{\prime} C}<0$ và không đổi. Gọi $M$ là hình chiếu của $A^{\prime}$ lên $A C$.

(a) Chứng minh rằng tâm $I$ đường tròn ngoại tiếp tam giác $B C M$ thuộc một đường thẳng cố định.

(b) Gọi $N$ là hình chiếu của $A^{\prime}$ lên $A B$ và $K$ là giao điểm hai tiếp tuyến của $\left(A^{\prime} M N\right)$ tại $M, N$. Chứng minh rằng $K$ thuộc một đường thẳng cố định.

Lời giải. (a) Đặt $\overline{A^{\prime} B} \cdot \overline{A^{\prime} C}=-k^2$. Từ $E$ hạ $I E \perp A A^{\prime}$. Gọi $N^{\prime}, P$ lần lượt là giao điểm của $(B M N)$ với $A A^{\prime}$. Ta có:

$\quad\quad\quad\quad\quad\quad\quad\quad\overline{A M} \cdot \overline{A B}=\overline{A N^{\prime}} \cdot \overline{A P}=A A^{\prime 2}=\left(\overline{A A^{\prime}}+\overline{A^{\prime} N^{\prime}}\right)\left(\overline{A A^{\prime}}+\overline{A^{\prime} P}\right)$

Do đó,

$\quad\quad\quad\quad\quad\quad\quad\quad\quad\quad\quad \overline{A A^{\prime}}\left(\overline{A^{\prime} N^{\prime}}+\overline{A^{\prime} P}\right)=-\overline{A^{\prime} N^{\prime}} \cdot \overline{A^{\prime} P}=k^2$

Ta thu dược

$\quad\quad\quad\quad\quad\quad\quad\quad\quad\quad\quad\quad\quad\quad\quad 2 \overline{A^{\prime} E}=\frac{k^2}{\overline{A A^{\prime}}}$

Suy ra $E$ cố định. Vậy điểm $I$ chạy trên đường thẳng qua $E$ và vuông góc với $A A^{\prime}$ cố định.

(b) Gọi $F$ là trung điểm của $A A^{\prime}$, ta có $F$ là tâm đường tròn ngoại tiếp tứ giác $A N A^{\prime} M$. Gọi $Z$ là giao điểm của $M N$ và $A A^{\prime}$, ta có

$\quad\quad\quad\quad\quad\quad\quad\quad\quad\quad\quad \overline{Z A} \cdot \overline{Z A^{\prime}}=\overline{Z M} \cdot \overline{Z N^{\prime}}=-k^2 \text {. }$

Suy ra $Z$ cố định.

Bây giờ, từ $K$ hạ $K Y$ vuông góc với $A A^{\prime}$. Ta có $F, M, N, K, Y$ cùng nằm trên một đường tròn, suy ra

$\quad\quad\quad\quad\quad\quad\quad\quad\quad\quad\quad \overline{Z F} \cdot \overline{Z Y}=\overline{Z M} \cdot \overline{Z N^{\prime}}=-k^2 .$

Từ đây dễ thấy $Y$ cố định. Vậy $K$ di động trên đường thẳng qua $Y$ vuông góc với $A A^{\prime}$ cố định.

Bài 4. Cho $f(x)=x^2+a x+b$ là tam thức bậc hai với $a, b \in \mathbb{R}$ và $f(f(x))=0$ có 4 nghiệm thực phân biệt. Biết rằng tổng của 2 nghiệm nào đó trong số 4 nghiệm đã nêu bằng $-1$. Chứng minh rằng $b \leq-\frac{1}{4}$.

Lời giải. Trước hết, dễ thấy $f(x)=0$ phải có 2 nghiệm phân biệt, đặt là $c_1<c_2$. Gọi $x_1, x_2$ là 2 trong số 4 nghiệm có tổng bằng $-1$. Theo định lý Viete thì: $c_1+c_2=$ $-a, c_1 c_2=b$. Ngoài ra,

$\quad\quad\quad\quad\quad\quad\quad\quad\quad\quad\quad f(f(x))=0 \Leftrightarrow f(x)=c_1 \vee f(x)=c_2 .$

Ta xét các trường hợp sau

  • Nếu $x_1, x_2$ là nghiệm của cùng một phương trình trong hai phương trình trên, theo định lý Viete thì $-a=x_1+x_2=-1$ nên $a=1$. Do $f(x)=c_1, f(x)=c_2$ đều phải có 2 nghiệm phân biệt nên $\Delta_1>0, \Delta_2>0$, tức là $1-4\left(b-c_1\right)>$ $0,1-4\left(b-c_2\right)>0$. Cộng lại, ta có

$\quad\quad\quad\quad\quad\quad\quad\quad\quad\quad\quad 2-4\left(2 b-c_1-c_2\right)>0 \Leftrightarrow 1-2(2 b+1)>0 \Leftrightarrow b<-\frac{1}{4} .$

  • Nếu $x_1, x_2$ là nghiệm của hai phương trình thì $x_1^2+a x_1+b=c_1, x_2^2+a x_2+b=c_2$. Cộng lại, ta có $x_1^2+x_2^2+a\left(x_1+x_2\right)+2 b=c_1+c_2 \Leftrightarrow x_1^2+x_2^2+2 b=0$. Do đó $b=-\frac{x_1^2+x_2^2}{2} \leq-\frac{\left(x_1+x_2\right)^2}{4}=-\frac{1}{4}$.

Trong mọi trường hợp, ta luôn có điều phải chứng minh.

Ngày thi thứ hai

Bài 5. Giả sử $P(x)=(x+1)^p(x-3)^q=x^n+a_1 x^{n-1}+a_2 x^{n-2}+\cdots+a_n$, trong đó $p, q$ là các số nguyên dương. Chứng minh rằng nếu $a_1=a_2$ thì $3 n$ là một số chính phương.

Lời giải. Ta có

$P(x)=(x+1)^p(x-3)^q=\left(x^p+C_p^1 x^{p-1}+C_p^2 x^{p-2}+\cdots\right)\left(x^q-3 C_q^1 x^{q-1}+9 C_q^2 x^{q-2}+\cdots\right)$

Từ đó suy ra

$\quad\quad\quad\quad\quad\quad\quad\quad\quad\quad\quad a_1=C_p^1-3 C_q^1 \text { và } a_2=C_p^2+9 C_q^2-3 C_p^1 C_q^1$

Như vậy $a_1=a_2$ khi và chỉ khi

$\quad\quad\quad\quad\quad\quad\quad\quad\quad\quad\quad p-3 q=\frac{p(p-1)}{2}+\frac{9 q(q-1)}{2}-3 p q$

hay

$\quad\quad\quad\quad\quad\quad 2 p-6 q=p^2-p+9 q^2-9 q-6 p q \text { tức là } 3 n=3(p+q)=(p-3 q)^2 \text {. }$

Suy ra $3 n$ là số chính phương. Ta có điều phải chứng minh.

Bài 6.

(a) Cho $a, b, c$ là các số thực dương. Chứng minh rằng ta có bất đẳng thức

$\quad\quad\quad\quad\quad\quad\quad\quad\quad\quad\quad \frac{a^2+b^2+c^2}{a b+b c+c a}+\frac{8 a b c}{(a+b)(b+c)(c+a)} \geq 2 .$

(b) Chứng minh rằng tồn tại các số thực dương $a, b, c$ sao cho

$\quad\quad\quad\quad\quad\quad\quad\quad\quad\quad\quad \frac{a b+b c+c a}{a^2+b^2+c^2}+\frac{(a+b)(b+c)(c+a)}{8 a b c}<2 .$

Lời giải. (a) Không mất tính tổng quát, ta có thể giả sử $c=\min {a, b, c}$. Khi đó, với chú ý rằng $a^2+b^2+c^2 \geq a b+b c+c a$, ta có

$\quad\quad\quad\quad\quad\quad\quad\quad\quad\quad \frac{a^2+b^2+c^2}{a b+b c+c a} =1+\frac{a^2+b^2+c^2-a b-b c-c a}{a b+b c+c a} $

$\quad\quad\quad\quad\quad\quad\quad\quad\quad\quad\quad\quad\quad\quad \geq 1+\frac{a^2+b^2+c^2-a b-b c-c a}{a b+b c+c a+c^2} $

$\quad\quad\quad\quad\quad\quad\quad\quad\quad\quad\quad\quad\quad\quad =\frac{a^2+b^2+2 c^2}{a b+b c+c a+c^2} $

$\quad\quad\quad\quad\quad\quad\quad\quad\quad\quad\quad\quad\quad\quad =\frac{a^2+b^2+2 c^2}{(a+c)(b+c)}$

Do đó, ta chỉ cần chứng minh.

$\quad\quad\quad \frac{a^2+b^2+2 c^2}{(a+c)(b+c)}+\frac{8 a b c}{(a+b)(b+c)(c+a)} \geq 2$

Bất đẳng thức này tương đương với

$a^3+b^3+a^2 b+b^2 a+2 c^2 a+2 c^2 b+8 a b c \geq 2\left(a^2 b+a^2 c+b^2 a+b^2 c+c^2 a+c^2 b+2 a b c\right)$

hay

$a^3+b^3+4 a b c \geq a^2 b+b^2 a+2 a^2 c+2 b^2 c . \Leftrightarrow(a-b)^2(a+b-2 c) \geq 0 .$

Do $c=min (a, b, c)$ nên bất đẳng thức cuối hiển nhiên đúng.

Phép chứng minh hoàn tất. Đẳng thức xảy ra khi và chỉ khi $a=b=c$.

(b) Kiểm tra trực tiếp, ta thấy bộ $(a, b, c)=(2,1,1)$ thỏa mãn đề bài. Điều này cho thấy rằng nếu nghịch đảo cả hai phân số trong vế trái của câu a) thì bài toán không còn đúng nữa.

Bài 7. Cho góc $O x y$ và một điểm $P$ nằm bên trong nó. Gọi $\gamma$ là đường tròn thay đổi nhưng luôn qua $O$ và $P$. Giả sử $\gamma$ cắt $O x, O y$ tại $M, N$. Tìm quỹ tích trọng tâm $G$ và trực tâm $H$ của tam giác $O M N$.

Lời giải. (a) Quĩ tích trọng tâm G của tam giác $O M N$

Gọi $I$ là trung điểm $M N$. Ta có ${G}=V_O^{\frac{2}{3}}({I})$. Ta sẽ tìm quỹ tích điểm $I$.

Phần thuận. Gọi $X$ là giao điểm thứ hai của $(I M P)$ với $O x, Y$ là giao điểm thứ hai của $(I N P)$ với $O y$. Ta có:

$\quad\quad\quad\quad\quad\quad\quad\quad\quad\quad\quad \angle X I P=\angle X M P=\angle P N Y=180^{\circ}-\angle P I Y$

do đó $X, Y, Z$ thẳng hàng. Mặt khác,

$\quad\quad\quad\quad\quad\quad\quad\quad\quad\quad\quad \angle I X P=\angle N M P=\angle P O N$

nên $\angle I X P$ không đổi. Tương tự ta cũng có $\angle X Y P$ không đổi mà $P$ cố định suy ra $X, Y$ cố định vậy $I$ nằm trên đường thẳng $X Y$ cố định.

Phần đảo. Lấy $X, Y$ lần lượt thuộc tia $O x, O y$ sao cho $\angle P X Y=\angle P O y$ và $\angle P Y X=$ $\angle P O x$. Lấy $I \in X Y$ ta sẽ chứng minh tồn tại $M \in O x, N \in O y$ sao cho $(O M N)$ đi qua $P$. Thật vậy,

Gọi $M$ là giao điểm thứ hai của $(I X P)$ và $O x, N$ là giao điểm thứ hai của $(I Y P)$ và $O y$. Ta có:

$\quad\quad\quad\quad\quad\quad\quad\quad\quad\quad\quad \angle X M P=\angle X I P=\angle P N Y$

nên tứ giác $O M P N$ nội tiếp. Ta có điều phải chứng minh.

(b) Quỹ tích trục tâm H của tam giác OMN.

Phần thuận. Gọi $T$ là trung điểm $O P . X, Y$ là hình chiếu của $P$ lên $O x, O y . K$ là trực tâm tam giác $O X Y . I$ là tâm đường tròn $(O M N)$. Ta có

$\quad\quad\quad\quad\quad\quad\quad\quad\quad\frac{O K}{O T}=\frac{O H}{O I}(=2|\cos \angle x O y|) \text { và } \angle I O T=\angle H O K$

nên $\triangle I O T \sim \triangle H O K$. Mà $\angle I T O=90^{\circ}$ nên $\angle H K O=90^{\circ}$. Vậy $H$ thuộc đường thẳng qua $K$ vuông góc với $O K$ (Chú ý $K$ cố định).

Phần đảo. Lấy $H$ thuộc đường thẳng qua $K$ vuông góc với $O K$, trong đó $K$ là trực tâm tam giác $O X Y$ và $X, Y$ là hình chiếu của $P$ lên $O x, O y$. Ta sẽ chứng minh tồn tại $M \in O x, N \in O y$ sao cho $(O M N)$ đi qua $P$ và $\triangle O M N$ nhận $H$ là trực tâm. Thật vậy, gọi $T$ là trung điểm $O P$ và dựng $\triangle O T I \sim \triangle O K H$. Ta có

$\quad\quad\quad\quad\quad\quad\quad\quad\quad\quad\quad \angle O T I=\angle O K H=90^{\circ}$

$\quad\quad\quad\quad\quad\quad\quad\quad\quad\quad\quad \angle I O T=\angle H O K$

nên $I$ nằm trên trung trực của $O P$. Do đó nếu vẽ $(T, T O)$ thì đường tròn này đi qua $P$ và cắt $O x, O y$ tại $M, N$. Ta có $K, T$ là trực tâm và tâm đường tròn $(O X Y)$ nên $\angle K O y=\angle T O x$. Ta được

$\quad\quad\quad\quad\quad\quad\quad\quad\quad\quad\quad \angle I O y=\angle H O x$

Lại có $\triangle O T I \sim \triangle O K H$ nên $\frac{O H}{O I}=\frac{O T}{O K}=2|\cos \angle x O y|$. Ta suy ra $H$ là trực tâm tam giác $O M N$

Bài 8. Với mỗi số nguyên dương gọi là tổng các chữ số của $n$.

(a) Chứng minh rằng $n=999, n=2999$ không thể biểu diễn thành tổng $a+b$ mà $S(a)=S(b)$.

(b) Chứng minh rằng với mọi $n$ mà $999<n<2999$ thì điều kiện trên được thỏa mãn.

Lời giải. Ta sẽ giải bài toán tồng quát sau: Tồn tại các số $a, b$ thỏa mãn điều kiện khi và chỉ khi số $n>1$ không có dạng $n=\overline{m 999 \ldots 9}$ với $0 \leq m \leq 8$ và $S(n)$ lẻ.

Thật vậy, xét $n=\overline{d_1 d_2 d_3 \ldots d_k}$ với $k$ là số các chữ số của $n$.

Ta có 2 trường hợp cần xét như sau:

  1. Nếu $S(n)$ chẵn (tương đương với có chẵn chữ số lẻ trong trong $n$ ), ta thực hiện như sau:
  • Nếu $d_i$ chẵn thì tách thành $d_i=\frac{d_i}{2}+\frac{d_i}{2}$ và chữ số ở hàng tương ứng của $a, b$ sẽ là 2 số này.
  • Nếu $d_i$ lẻ thì tách thành $d_i=\frac{d_i-1}{2}+\frac{d_i+1}{2}$ thì $\frac{d_i+1}{2}-\frac{d_i-1}{2}=1$ và như thế, ta luân phiên thay đổi các số lớn nhỏ để ghép vào $a, b$ để đảm bảo có $S(a)=S(b)$.

Do có chẵn số $d_i$ lẻ như thế nên quá trình trên thực hiện được và trong trường hợp này, tồn tại số $a, b$ thỏa mãn.

  1. Nếu $S(n)$ lẻ (tương đương với có lẻ chữ số lẻ trong trong $n$ ). Nếu $n$ có dạng $m 999 \ldots 9$ với $0 \leq m \leq 8$ thì rõ ràng khi tách ra thành 2 phần, các phép tính tồng phía sau để thu được các sồ 9 là không có nhớ và chúng có dạng $d_i+d_i^{\prime}=9$. Khi đó, ta có

$\quad\quad\quad\quad\quad\quad\quad S(a)+S(b)=S(n) \text { mà }\left\{\begin{array}{l}S(a)=S(b) \\ S(n) \equiv 1(\bmod 2)\end{array}\right.$

Điều mâu thuân trên cho thấy trường hợp này không tồn tại cách tách $n$ thành $a, b$ thỏa mãn. Nếu $n$ không có dạng trên thì dễ thấy tồn tại $d_i \neq 0, d_{i+1} \neq 9$.

Ta viết lại $n$ như sau:

$n=\overline{d_1 d_2 d_3 \ldots d_i d_{i+1} \ldots d_k}=\overline{d_1 d_2 d_{3 \ldots}\left(d_i-1\right) 9 \ldots d_k}+\left(d_{i+1}+1\right) 10^{k-d_i}$

Đặt $a=\overline{d_1 d_2 d_3 \ldots\left(d_i-1\right) 9 \ldots d_k}, b=\left(d_{i+1}+1\right) 10^{k-d_i}$ thì $S(a), S(b)$ cùng tính chẵn lẻ và $a+b=n$. Ta có thể giả sử $S(a)>S(b)$, trường hợp còn lại chứng minh tương tự. Nếu chọn một vị trí $t$ mà $d_t \neq 0$ thì có thể đổi các số $a, b$ thành

$a^{\prime}=\overline{d_1 d_2 d_3 \ldots\left(d_i-1\right) 9 \ldots\left(d_t-1\right) \ldots d_k}, y=\left(d_{i+1}+1\right) 10^{k-i}+10^{k-t}$

với $k \neq i$. Khi đó

$\quad\quad\quad \quad\quad\quad S\left(a^{\prime}\right)-S\left(b^{\prime}\right)=S(a)-1-(S(b)+1)=-2 .$

Cứ như vậy, ta thực hiện liên tiếp đến khi nào chênh lệch giữa hai tổng các chữ số bằng 0 thì dừng lại (tồn tại thời điểm như vậy vì ban đầu chúng cùng tính chẵn lẻ nên hiệu của chúng là số chẵn và mỗi lần thực hiện quá trình trên thì hiệu giảm đi 2 đơn vị). Các số $a, b$ lúc đó sẽ thỏa mãn đề bài và cũng tồn tại cách tách.

Bài toán được giải quyết hoàn toàn.

 

 

 

 

 

 

 

 

 

 

 

 

 

 

 

 

 

 

 

 

 

 

 

 

 

 

 

 

 

 

 

 

 

 

Đề thi và đáp án chọn đội tuyển toán trường PTNK năm 2021

Ngày thi thứ nhất. 

Bài 1. Tìm hàm số $f: \mathbb{R} \rightarrow \mathbb{R}$ thỏa $f(x f(y)+f(x))=f(x)+x y+x+1, \forall x, y \in \mathbb{R} .$

Bài 2. Cho dãy số $\left(u_{n}\right)$ thỏa $u_{1}=2, u_{2}=1$ và $u_{n+1}=\sqrt{\dfrac{u_{n} u_{n-1}}{n}}$ với mọi $n \geq 2$.
Xét dãy số $\left(v_{n}\right)$ xác định bởi $v_{n}:=u_{1}+u_{2}+\ldots+u_{n}, \forall n \geq 1$. Chứng minh dãy $\left(v_{n}\right)$ hội tụ.

Bài 3. Cho $p$ là số nguyên tố, $n$ là số nguyên dương thỏa $2<p<n$. Gọi $\mathrm{A}$ là tập hợp các đa thức $P(x)=x^{n}+a_{n-1} x^{n-1}+\ldots+a_{1} x+a_{0}$ có tất cả các hệ số thuộc tập ${1 ; 2 ; \ldots ; n !}$ và $P(m)$ chia hết cho $p$ với mọi số nguyên dương $m$.

a) Chứng minh tổng $a_{1}+a_{p}+a_{2 p-1}+\ldots+a_{1+k(p-1)}$ chia hết cho $p$ với mọi $k=\left[\dfrac{n-1}{p-1}\right]$ (xem $a_{n}=1$ ), kí hiệu $[x]$ là phần nguyên của $x$.
b) Tính số phần tử của $\mathrm{A}$ theo $\mathrm{n}$ và $\mathrm{p}$.

Bài 4. Cho tam giác $\mathrm{ABC}$ có (I) là đường tròn nội tiếp. Một đường thẳng qua $\mathrm{A}$ cắt $(\mathrm{I})$ tại $\mathrm{M}, \mathrm{N}$. Gọi $\mathrm{T}$ là giao điểm của các tiếp tuyến với (I) tại $\mathrm{M}, \mathrm{N}$.

b) Chứng minh rằng nếu $\mathrm{AT} \parallel \mathrm{BC}$ thì $\mathrm{MN}$ đi qua trung điểm $\mathrm{K}$ của $\mathrm{BC}$.
c) Gọi $\mathrm{D}$ là tiếp điểm của (I) với $\mathrm{AB}$ và $\mathrm{E}$ là giao điểm của $\mathrm{DM}$ với $\mathrm{AC}$. Trên $\mathrm{EN}$ lấy điểm $\mathrm{F}$ thoả $\mathrm{TF}$ vuông góc $\mathrm{AI}$. Chứng minh rằng khi đường thẳng $\mathrm{AMN}$ thay đổi, giao điểm $\mathrm{P}$ của $\mathrm{MF}$ và $\mathrm{DN}$ thuộc một đường thẳng cố định.

Ngày thi thứ hai

Bài 5. Cho $n$ số thực $x_{1}, x_{2}, \ldots, x_{n}$ thỏa hiệu giữa số lớn nhất và số nhỏ nhất của chúng là 1 . Ta xây dựng
$$
y_{1}=x_{1}, y_{2}=\frac{x_{1}+x_{2}}{2}, \ldots, y_{n}=\frac{x_{1}+x_{2}+\ldots+x_{n}}{n}
$$
Gọi $M, m$ lần lượt là số lớn nhất và nhỏ nhất trong các số $y_1, y_2,\cdots,y_n$. \
Tìm giá trị lớn nhất của $M-n$.

Bài 6.  Cho tập $\mathrm{X}={1 ; 2 ; \ldots ; 20}$. Tập con $\mathrm{A}$ của $\mathrm{X}$ được gọi là tập “tránh 2 ” nếu với mọi $\mathrm{x}, \mathrm{y}$ thuộc $\mathrm{A}$ thì $|x-y|$ khác 2 . Tìm số các tập con “tránh 2 ” của $\mathrm{X}$ có 5 phần tử.

Bài 7. Cho tam giác $\mathrm{ABC}$ và điểm $\mathrm{D}$ trên cạnh $\mathrm{BC}$. Các đường tròn ( $\mathrm{ABD}$ ), ( $\mathrm{ACD}$ ) lần lượt cắt $\mathrm{AC}, \mathrm{AB}$ tại $\mathrm{E}, \mathrm{F}$. Gọi $\mathrm{I}$ là tâm đường tròn $(\mathrm{AEF})$.
a) Chứng minh ID vuông góc BC.
b) Gọi $\mathrm{H}$ là giao điểm của $\mathrm{ID}$ với $\mathrm{EF}$ và $\mathrm{K}$ là điểm thoả mãn $H B K=H C K=90^{\circ}$. Các đường tròn (IBK), (ICK) lần lượt cắt IC, IB tại M, N. Chứng minh tâm J của đường tròn (IMN) thuộc trung trực BC.

Bài 8.  Cho $p$ là số nguyên tố. Với mọi số nguyên a, đặt
$$
q:=1+a+a^{2}+\ldots+a^{p-1} .
$$
Chứng minh $(1-a)\left(1-a^{2}\right) \ldots\left(1-a^{p-1}\right)-p$ chia hết cho $q$.

 

Đáp án sẽ được đăng trong Tập san Star education số 7/2022

Đáp án đề thi chọn đội tuyển trường PTNK năm 2020

Ngày thi thứ nhất.

Bài 1. Với mỗi số nguyên dương $n$, tìm số thực $M_{n}$ lớn nhất sao cho với mọi số thực dương $x_{1}, x_{2}, \ldots, x_{n}$ thì ta đều có
$$
\sum_{k=1}^{n} \frac{1}{x_{k}^{2}}+\frac{1}{\left(\sum_{k=1}^{n} x_{k}\right)^{2}} \geq M_{n}\left(\sum_{k=1}^{n} \frac{1}{x_{k}}+\frac{1}{\sum_{k=1}^{n} x_{k}}\right)^{2}
$$

Bài 2. Cho 2021 số nguyên khác 0 . Biết rằng tổng của một số bất kỳ trong chúng với tích của tất cả 2020 số còn lại luôn âm.
(a) Chứng minh rằng với mọi cách chia 2021 số này thành hai nhóm và nhân các số cùng nhóm lại với nhau thì tổng của hai tích cũng luôn âm.
(b) Một bộ số thỏa mãn đề bài thì có thể có nhiều nhất mấy số âm?

Bài 3. Cho hai hàm số $f: \mathbb{R} \rightarrow \mathbb{R}$ và $g: \mathbb{R} \rightarrow \mathbb{R}$ thỏa mãn $g(2020)>0$ và với mọi $x, y \in \mathbb{R}$ thì $\left\{\begin{array}{l}f(x-g(y))=f(-x+2 g(y))+x g(y)-6 \\ g(y)=g(2 f(x)-y)\end{array}\right.$

(a) Chứng minh rằng $g$ là hàm hằng.

(b) Chứng minh rằng đồ thị $h(x)=f(x)-x$ nhận $x=1$ là trục đối xứng.

Bài 4. Cho tam giác $A B C$ nhọn, nội tiếp trong đường tròn $(O)$ có trực tâm $H$ và $A H, B H, C H$ cắt cạnh đối diện lần lượt tại $D, E, F$. Gọi $I, M, N$ lần lượt là trung điểm các cạnh $B C, H B, H C$ và $B H, C H$ cắt lại $(O)$ theo thứ tự tại các diểm $L, K$. Giả sử $K L$ cắt $M N$ ở $G$.
(a) Trên $E F$, lấy điểm $T$ sao cho $A T$ vuông góc với $H I$. Chứng minh rằng $G T$ vuông góc với $O H$.
(b) Gọi $P, Q$ lần lượt là giao điểm của $D E, D F$ và $M N$. Gọi $S$ là giao điểm của $B Q, C P$. Chứng minh rằng $H S$ di qua trung điểm của $E F$.

Ngày thi thứ hai.
Bài 5. Cho số nguyên dương $n>1$. Chứng minh rằng với mọi số thực $a \in\left(0 ; \frac{1}{n}\right)$ và mọi đa thức $P(x)$ có bậc $2 n-1$ thỏa mãn điều kiện $P(0)=P(1)=0$, luôn tồn tại các số thực $x_{1}, x_{2}$ thuộc $[0 ; 1]$ sao cho $P\left(x_{1}\right)=P\left(x_{2}\right)$ và $x_{2}-x_{1}=a$.

Bài 6. Giải phương trình sau trên $\mathbb{Z}^{+}:\left(x^{2}+3\right)^{3^{x+1}}\left[\left(x^{2}+3\right)^{3^{x+1}}+1\right]+x^{2}+y=x^{2} y$.

Bài 7 . Cho các số nguyên $n>k>t>0$ và $X={1,2, \ldots, n}$. Gọi $\mathcal{F}$ là họ các tập con có $k$ phần tử của tập hợp $X$ sao cho với mọi $F, F^{\prime} \in \mathcal{F}$ thì $\left|F \cap F^{\prime}\right| \geq t$. Giả sử không có tập con có $t$ phần tử nào chứa trong tất cả các tập $F \in \mathcal{F}$.
(a) Chứng minh rằng tồn tại một tập hợp $B \subset X$ sao cho $|B|<3 k$ và $|B \cap F| \geq t+1$ với mọi $F \in \mathcal{F}$.
(b) Chứng minh rằng $|\mathcal{F}|<C_{3 k}^{t+1} C_{n}^{k-t-1}$.

Bài 8. Cho tam giác $A B C$ nội tiếp trong $(O)$ với $B, C$ cố định và $A$ thay đổi trên cung lớn $B C$. Dựng hình bình hành $A B D C$ và $A D$ cắt lại $(B C D)$ ở $K$.
(a) Gọi $R_{1}, R_{2}$ lần lượt là bán kính đường tròn ngoại tiếp $(K A B),(K A C)$. Chứng minh rằng tích $R_{1} R_{2}$ không đổi.
(b) Ký hiệu $(T),\left(T^{\prime}\right)$ lần lượt là các đường tròn cùng đi qua $K$, tiếp xúc với $B D$ ở $B$ và tiếp xúc với $C D$ ở $C$. Giả sử $(T),\left(T^{\prime}\right)$ cắt nhau ở $L \neq K$. Chứng minh rằng $A L$ luôn đi qua một điểm cố định.

Hết

Đề thi và đáp án kì thi chọn đội tuyển thi Quốc gia trường Phổ thông Năng khiếu năm học 2016 -2017

Đề thi

Ngày thi thứ nhất

Bài 1. Tìm tất cả $a$ để dãy số $(u_n)$ hội tụ, biết $u_1=a$ và $\forall n\in \mathbb{N}^*$ thì:
$$u_{n+1}=\left\{\begin{array}{l}
2u_n-1\ \text{nếu $u_n>0$,}\\
-1\ \text{nếu $-1\le u_n\le 0$,}\\
u_n^2+4u_n+2\ \text{nếu $u_n<-1$.}
\end{array} \right.$$

Bài 2. Tìm số nguyên dương $k$ nhỏ nhất để bất đẳng thức $$x^ky^kz^k(x^3+y^3+z^3)\le 3$$
luôn đúng với mọi số thực dương $x,y,z$ thoả mãn điều kiện $x+y+z=3$.

Bài 3. Cho hàm số $f:\mathbb N^* \rightarrow \mathbb N^*$ thoả mãn hai điều kiện sau:

i)  $f$ là hàm tăng thật sự trên $\mathbb N^*$.

ii) $f(2n)=2f(n)\ \forall n\in \mathbb N^*$.

a) Giả sử $f(1)=3$ và $p>3$ là số nguyên tố. Chứng minh rằng tồn tại số nguyên dương $n$ sao cho $f(n)$ chia hết cho $p$.
b) Cho $q$ là số nguyên tố lẻ. Hãy xây dựng một hàm $f$ thoả mãn các điều kiện của bài toán mà $f(n)$ không chia hết cho $q$ với mọi $n$ nguyên dương.

Bài 4. Cho tam giác $ABC$ có góc $\angle BAC$ tù và $AH\perp BC$ ($H$ nằm trên $BC$). Điểm $M$ thay đổi trên cạnh $AB$. Dựng điểm $N$ sao cho $\Delta BMN\sim \Delta HCA$, với $H$ và $N$ nằm khác phía đối với đường thẳng $AB$.

a) Gọi $CM$ cắt đường tròn ngoại tiếp tam giác $BMN$ tại $K$. Chứng minh rằng $NK$ luôn đi qua một điểm cố định.
b) Gọi $NH$ cắt $AC$ tại $P$. Dựng điểm $Q$ sao cho $\triangle HPQ\sim \Delta HNM$, với $Q$ và $M$ nằm khác phía đối với đường thẳng $NP$. Chứng minh rằng $Q$ luôn thuộc một đường thẳng cố định.

Ngày thi thứ hai

Bài 5. Với mỗi số nguyên dương $n$, tồn tại duy nhất số tự nhiên $a$ thoả mãn điều kiện $a^2\le n<(a+1)^2$. Đặt $\Delta_n=n-a^2$.

a) Tìm giá trị nhỏ nhất của $\Delta_n$ khi $n$ thay đổi và luôn thoả mãn $n=15m^2$ với $m$ là số nguyên dương.
b) Cho $p,q$ là các số nguyên dương và $d=5(4p+3)q^2$. Chứng minh rằng $\Delta_d\ge 5$.

Bài 6.  Với các số nguyên $a,b,c,d$ thoả mãn $1\le a<b<c<d$, ký hiệu:
$$T(a,b,c,d)={{x,y,z,t}\subset \mathbb{N}^*\mid 1\le x<y<z<t,\ x\le a,y\le b,z\le c,t\le d}$$

a) Tình số phần tử của $T(1,4,6,7)$.
b) Cho $a=1$ và $b\ge 4$. Gọi $d_1$ là số phần tử của $T(a,b,c,d)$ chứa $1$ và không chứa $2$; $d_2$ là số phần tử chứa $1,2$ và không chứa $3$; $d_3$ là số phần tử chứa $1,2,3$ và không chứa $4$. Chứng minh rằng $d_1\ge 2d_2-d_3$. Đẳng thức xảy ra khi nào ?

Bài 7. Trong một hệ thống máy tính, một máy tính bất kỳ có kết nối trực tiếp với ít nhất $30\%$ máy tính khác của hệ thống. Hệ thống này có một chương trình cảnh báo và ngăn chặn khá tốt, do đó khi một máy tính bị virus, nó chỉ có đủ thời gian lây cho các máy tính được kết nối trực tiếp với nó. Chứng minh rằng dù vậy, kẻ tấn công vẫn có thể chọn hai máy tính của hệ thống mà nếu thả virus vào hai máy đó, ít nhất $50\%$ máy tính của hệ thống sẽ bị nhiễm virus.

Bài 8. Cho tam giác $ABC$ nhọn. Đường tròn $(I)$ có tâm $I$ thuộc cạnh $BC$ và tiếp xúc với các cạnh $AB,AC$ lần lượt tại $E,F$. Lấy $M,N$ bên trong tứ giác $BCEF$ sao cho $EFNM$ nội tiếp $(I)$ và các đường thẳng $MN,EF,BC$ đồng quy. Gọi $MF$ cắt $NE$ tại $P$, $AP$ cắt $BC$ tại $D$.

a) Chứng minh rằng $A,D,E,F$ cùng thuộc một đường tròn.
b) Lấy trên các đường thẳng $BN,CM$ các điểm $H,K$ sao cho $\angle ACH=\angle ABK=90^\circ$. Gọi $T$ là trung điểm $HK$. Chứng minh rằng $TB=TC$.

Hết

Lời giải

Lời giải

Bài 1. 

  • Nếu $a>1$, bằng quy nạp đơn giản, ta có $u_n>1\ \forall n\in \mathbb N^*$ và
    $$u_n = 2^{n-1}(a-1)+1, \ \forall n\in \mathbb N^*.$$
    Do $a>1$, cho $n\rightarrow +\infty$ thì $u_n\rightarrow +\infty$. Từ đó $(u_n)$ không hội tụ.
  •  Nếu $a=1$ thì $u_n=1\ \forall n\in \mathbb N^*$ hay $(u_n)$ hội tụ về $1$.
  • Nếu $0<a<1$, ta sẽ chứng minh rằng $(u_n)$ có ít nhất một số hạng không dương. Thật vậy, giả sử $u_n>0\ \forall n\in \mathbb N^*$ thì theo trường hợp đầu tiên, ta có:
    $$u_n = 2^{n-1}(a-1)+1\ \forall n\in \mathbb N^*$$
    Do $a>1$, cho $n\rightarrow +\infty$ thì $u_n\rightarrow -\infty$, trái với việc $u_n>0\ \forall n, \in \mathbb N^*$. Từ đó điều giả sử là sai hay phải tồn tại $k\in \mathbb N^*$ sao cho $u_k>0$ và $u_{k+1}\le 0$. Với cách chọn chỉ số $k$ như vậy, ta có:
    $$-1\le 2u_k-1=u_{k+1}\le 0$$
    Khi đó $u_{k+2}=0$. Bằng quy nạp thì $u_n=-1\ \forall n\in \mathbb N^*, n\ge k+2$. Điều này dẫn đến $(u_n)$ hội tụ về $-1$.
  • Nếu $-1\le a\le 0$, từ giả thiết thì $u_2=-1$. Bằng quy nạp thì $u_n=-1\ \forall n\in \mathbb N^*, n\ge 2$ hay $(u_n)$ hội tụ về $-1$.
  • Nếu $-2<a<-1$, ta có:
    $$u_2-u_1=a^2+3a+2=(a+2)(a+1)<0$$
    Khi đó thì $u_2<u_1<-1$. Lại có $u_2=(a+2)^2-2\ge -2$ nên $-2<u_2<-1$. Bằng quy nạp, ta có $(u_n)$ là dãy giảm và $-2<u_n<-1$ nên $(u_n)$ hội tụ.
  • Nếu $-2-\sqrt{3}\le a\le -2$ thì $u_2=a^2-4a+2$ và dễ có được:
    $$-1\le a^2-4a+2\le 1$$
    Theo các trường hợp đã xét, dãy số $(u_n)$ hội tụ.
  • Nếu $a<-2-\sqrt{3}$, bằng vài tính toán, ta có $u^2=a^2-4a+2>1$.\\
    Theo trường hợp đầu tiên, dãy số $(u_n)$ không hội tụ.Vậy dãy số $(u_n)$ hội tụ khi và chỉ khi $-2-\sqrt{3}\le a\le 1$.]

Bài 2. Ta sẽ chứng minh rằng $k=3$ là số nguyên dương nhỏ nhất thoả mãn bài toán. Trước hết, chọn $x=y=\dfrac{3}{4},z=\dfrac{3}{2}$ thì ta phải có:
$$\left(\frac{3}{4}\right)^{2k}\cdot \left(\frac{3}{2}\right)^{k}\left(2\cdot\left(\frac{3}{4}\right)^{3}+\left(\frac{3}{2}\right)^3\right)\le 3$$
Dễ thấy đánh giá trên chỉ đúng nếu $k\ge 3$. Ta đưa về chứng minh rằng:
$$x^3y^3z^3(x^3+y^3+z^3)\le 3.$$
Không mất tính tổng quát, giả sử $x\ge y\ge z$ thì $z \le 1$. Ta có:
$$\begin{aligned} & x^3+y^3=(x+y)^3-3xy(x+y)=(3-z)^3-3xy(x+y) \text{ hay} \\
&(3-z)^3 + z^3 \le \frac{3}{x^3y^3z^3}+3xy(x+y). \end{aligned} $$
Khai triển và thu gọn, bất đẳng thức trở thành:
$$3z^2-9z+9 \le \frac{1}{x^3y^3z^3}+x^2y+xy^2.$$
Theo bất đẳng thức AM-GM, ta có vế phải của bất đẳng thức trên sẽ không nhỏ hơn $\frac{3}{z}$. Từ đây ta chỉ cần chứng minh rằng $$3z^2-9z+9 \le \dfrac{3}{z} \text{ hay } 3(z-1)^3 \le 0, \text{ đúng.}$$
Vậy $k=3$ là hằng số nguyên dương nhỏ nhất thoả mãn bài toán.

Bài 3.

(a) Đặt $A = \\{f(n+1)-f(n)|n \in\mathbb{N^{*}}\\}$.

Vì $f$ là hàm số tăng thực sự trên $\mathbb{N^{*}}$ nên $A\subset \mathbb{N^{*}}$.

Khi đó phải tồn tại $k=\min A$ và tồn tại $n\in \mathbb N^*$ để $k=f(n+1)-f(n)$.

Khi đó:
$$f(2n+2) – f(2n) = 2f(n+1) – 2f(n) = 2k.$$

Lại có $f(2n+2) – f(2n+1),f(2n+1) – f(2n)\ge k$ nên

$$f(2n+2)-f(2n+1)+f(2n+1)-f(2n)\ge 2k.$$
Từ đây ta phải có $f(2n+2) – f(2n+1)=f(2n+1) – f(2n)= k$. Bằng quy nạp theo $m$, ta chứng minh được

$$f(2^mn+t) = 2^mf(n)+tk\ \forall t,m\in \mathbb N, t\le m.$$

Lại có $f(1)=3,f(2)=6$ nên $k \le 3<p$ hay $(k, p)=1$. \medskip

Xét $p$ số nguyên dương sau:
$$f(2^pn), f(2^pn+1), f(2^pn+2),\ldots, f(2^pn+p-1)$$
lập thành một cấp số cộng có công sai $k$ nên là một hệ thặng dư đầy đủ modulo $p$. Từ đó phải tồn tại một số hạng chia hết cho $p$. \medskip

(b) Ta xây dựng một hàm số $f$ với các điều kiện như sau:

$f(1) = 2^a > q (a\in\mathbb{N^{*}},$

$f(2n)=2f(n)\ \forall n\in \mathbb{N^{*}},$

$f(2n+1)=f(2n)+q\ \forall n\in \mathbb{N^{*}}.$

Ta chứng minh rằng hàm số $f$ vừa xây dựng thỏa mãn bài toán. \medskip

Trước hết ta chứng minh rằng $f$ là hàm tăng thực sự, cụ thể là:
$$f(n+1) – f(n) \geq q\ \forall n\in \mathbb{N^{*}}.$$
Với $n = 1$, ta có $f(2)-f(1) = 2.2^a – 2^a = 2^a > q$. Giả sử khẳng định cần chứng minh đúng đến $n=k$. Xét các khả năng sau:

Nếu $k$ là số chẵn, ta có $f(k+1)=f(k)+q$ thỏa mãn yêu cầu.
Nếu $k$ là số lẻ, ta có:
$$f(k+1)= 2f\left(\dfrac{k+1}{2}\right) \geq 2\left(f\left(\dfrac{k-1}{2}\right)+q\right)= f(k-1)+2q.$$
Lại có $f(k)=f(k-1)+q$ nên $f(k+1)\ge f(k)+q$.

Theo nguyên lý quy nạp, ta có $f(n+1) – f(n) \geq q\ \forall n\in \mathbb{N^{*}}$. \medskip

Bây giờ ta chứng minh rằng không tồn tại $n$ để $q \mid f(n)$. Trước hết thì $f(1) = 2^a$ không chia hết cho $q$. Giả sử điều này đúng đến $n=k$. Xét các khả năng sau:

Nếu $k$ chẵn thì $f(k+1)=f(k)+q$ không chia hết cho $q$.
Nếu $k$ lẻ thì $f(k+1)= 2f\left(\dfrac{k+1}{2}\right)$ không chia hết cho $q$.

Theo nguyên lý quy nạp, $f(n)$ không chia hết cho $q$ với mọi $n\in \mathbb{N^{*}}$.
Các điều kiện đã được kiểm tra đầy đủ.

BÀI 4.

(a) Ta sẽ chứng minh rằng $AD\perp BC$. Gọi $X$ là điểm đồng quy của $EF,MN,BC$. Do $AE,AF$ tiếp xúc với $(I)$ nên $EF$ là đường đối cực của $A$ đối với $(I)$. Ta có $X\in EF$ nên theo định lý La Hire, điểm $A$ sẽ nằm trên đường đối cực của $X$ đối với đường tròn $(I)$. \medskip

Lại có $P$ là giao điểm của $EN,FM$ nên $P$ nằm trên đường đối cực của $X$ đối với $(I)$. Vì thế nên $AP$ là đường đối cực của $X$ đối với $(I)$ hay $AP\perp BC$. Do đó $$\angle ADI=\angle AEI=\angle AFI=90^\circ.$$
Vậy $A,D,E,F$ cùng thuộc một đường tròn.

(b) Gọi $S$ là giao điểm của $BN,CM$. Xét hai tam giác $PEF,SBC$ có $PE$ cắt $SB$ tại $N$, $PF$ cắt $SC$ tại $M$, $EF$ cắt $BC$ tại $X$ và $X,M,N$ thẳng hàng. Theo định lý Desargues thì $PS,EB,FC$ đồng quy. Mặt khác $EB$ cắt $FC$ tại $A$ nên $A,P,S$ thẳng hàng, dẫn đến $S\in AD$. \medskip

Tiếp theo ta sẽ chứng minh rằng $\angle BAK=\angle CAH$. Áp dụng định lý Ceva dạng lượng giác cho tam giác $ABC$ với:

Các đường thẳng $AD,BH,CK$ đồng quy:
$$\frac{\sin\angle DAB}{\sin \angle DAC}\cdot \frac{\sin\angle HBC}{\sin \angle HBA}\cdot \frac{\sin\angle KCA}{\sin \angle KCB}=1$$
Các đường thẳng $AH,BH,CH$ đồng quy:
$$\frac{\sin\angle HAB}{\sin \angle HAC}\cdot \frac{\sin\angle HBC}{\sin \angle HBA}\cdot \frac{\sin\angle HCA}{\sin \angle HCB}=1$$
Các đường thẳng $AK,BK,CK$ đồng quy:
$$\frac{\sin\angle KAB}{\sin \angle KAC}\cdot \frac{\sin\angle KBC}{\sin \angle KBA}\cdot \frac{\sin\angle KCA}{\sin \angle KCB}=1$$

Chú ý rằng do các góc vuông và góc bù nhau nên ta có
$$\frac{\sin\angle HAC}{\sin \angle HAB}=\frac{\sin\angle KAB}{\sin \angle KAC}$$
Từ đó sử dụng công thức cộng cho mẫu thức và biến đổi thì:
$$\tan\angle HAC=\tan\angle KAB$$
Dẫn đến $\angle HAC=\angle KAB$. Cuối cùng, ta sẽ chứng minh $TB=TC$.
Gọi $U,V$ lần lượt là trung điểm của các đoạn $AK,AH$. Ta có:
$$UB=\dfrac{AK}{2}=VT,UT=\dfrac{AH}{2}=VC.$$
Đồng thời, ta cũng có:
$$\angle BUT=\angle BUA-\angle AUT=\angle AVC-\angle AVT=\angle TVC$$
Do đó $\Delta BUT=\Delta TVC$ (c.g.c), vậy nên $TB=TC$.

Bài 5. 

(a) Ta cần tìm $\Delta_n$ nhỏ nhất để phương trình $15m^2 – a^2 = \Delta_n$ có nghiệm nguyên dương. Nhận thấy $15 – 3^2 = 6$ nên $\min \Delta_n\le 6$. Ta chứng minh rằng phương trình trên không có nghiệm nguyên dương với $\Delta_n < 6$. \medskip

Ta có $3\mid a^2 + \Delta_n$. Suy ra $3\mid \Delta_n $ hoặc $3\mid \Delta_n+1$. Mặt khác $5\mid a^2 + \Delta_n$ nên $\Delta_n$ chia $5$ chỉ có thể dư $0,1$ hoặc $4$. \medskip

Từ đó nếu tồn tại $n$ để $\Delta_n< 6$ thỏa mãn bài toán thì $\Delta_n = 5$. Giả sử rằng tồn tại $n$ như thế, ta có $15m^2-a^2=5$ hay $5\mid a$. Đặt $a=5s$ $(s\in \mathbb N^*)$, ta có:
$$3m^2 – 5s^2 = 1.$$
Từ đó thì $$3(m^2+s^2)\equiv 1 \pmod{8} \text{ hay } m^2+s^2\equiv 3 \pmod{8}.$$
Điều này vô lý do $m^2$ chia $8$ dư $0,1,4$. Vậy $\Delta_n$ nhỏ nhất là $6.$ \medskip

(b) Ta có $$5(4p+3)q^2-a^2=\Delta_d.$$ Do $a^2$ chia $5$ dư $0,1,4$ nên $\Delta_d$ chia $5$ dư $0,1,4$. Giả sử rằng có bộ số để $\Delta_d<5$. Xét các khả năng sau:

Nếu $\Delta_d=0$ thì $5(4p+3)q^2=a^2$. Xét bộ số $(q,a)$ với $q+a$ nhỏ nhất. Từ phương trình trên, ta có $a^2+q^2\equiv 0$ (mod $4$) hay $a\equiv q\equiv 0$ (mod $2$).\medskip

Đặt $a=2a_1$ và $q=2q_1$ với $a_1,q_1\in \mathbb N^*$ thì bộ số $(q_1,a_1)$ cũng thoả mãn điều kiện $5(4p+3)q_1^2=a_1^2$. Hơn nữa $q_1+a_1<q+a$, mâu thuẫn.
Nếu $\Delta_d = 1$, ta có $a^2 + 1 = 5(4p+3)q^2$. Do $5(4p+3)\equiv 3$ (mod $4$) nên số này tồn tại một ước nguyên tố $r\equiv 3$ (mod $4$).\\
Do đó $a^2+1\equiv 0$ (mod $r$) hay $r\mid 1$, vô lý.
Nếu $\Delta_d = 4$, chứng minh tương tự, ta cũng có điều mâu thuẫn.

Vậy ta phải có $\Delta_d \ge 5$.

Bài 6.

(a) Với $T(1,4,6,7)$, ta có $x \leq 1$ nên $x =1$. Khi đó ta có $2\le y \le 4$ hay $y\in \{2,3,4\}$. Xét các khả năng sau:

Nếu $y = 2$ thì $3\leq z \leq 6$. Với mỗi giá trị của $z$, ta có thể thu được $7-z$ giá trị của $t$ nên ta có 10 bộ số.
Nếu $y=3$, tương tự ta có $6$ bộ số.
Nếu $y= 4$, tương tự ta có $3$ bộ số.

Vậy có tất cả $19$ bộ số trong $T(1,4,6,7)$. \medskip

(b) Đặt các tập hợp sau:
$$\begin{cases}
T_1 = \{(1, y, z, t)\mid 3\le y \le b, y<z\le c, z<t \leq d \}\\\\
T_2 = \{(1, 2, z, t)\mid 4\le z \le c, z<t\le d \}\\\\
T_3 = \{(1, 2, 3, t)\mid 5\le t \le d \}
\end{cases}.$$
Ta có $d_3 = |T_3| = d – 4$ và
$$d_2=\sum_{z=4}^{c}(d-z)=(c-3)d+\frac{(c+4)(c-3)}{2}.$$
Tiếp theo ta tính $d_1=|T_1|$. Vì $b \ge 4$ nên $y \ge 3$. Xét các khả năng sau

Nếu $y=3$ thì $T(1,3,z,t)=d_2$.
Nếu $y=4$ thì $T(1,4,z,t)=\sum_{z=5}^{c} (d-z)=(c-4)d-\dfrac{(c+5)(c-4)}{2}$. \medskip

Từ đó $d_1\ge d_2+(c-4)d-\dfrac{(c+5)(c-4)}{2}$. Do đó, kết hợp với việc tính được giá trị của $d_2$, khi cộng theo vế thì $d_1+d_3 – 2d_2 \ge 0.$

Vậy $d_1\ge 2d_2-d_3$. Đẳng thức xảy ra khi và chỉ khi $b=4$.

Ngoài lời giải khá “đại số” phía trên, có một lời giải khác cho ý sau của bài toán sử dụng song ánh:

Điểm mấu chốt là phân rã $T_1,T_2,T_3$ thành các nhóm thích hợp và thiết lập được đơn ánh giữa chúng. Với các tập $T_1,T_2,T_3$ định nghĩa như trên, ta viết $T_1$ thành $A\cup B\cup C$ có giao đôi một khác rỗng, trong đó
$$\begin{cases}
A = \{(1, 3, 4, t)\mid 5\le t \le d \}\\\\
B = \{(1, 3, z, t)\mid 5\le z \le c, z<t\le d \}\\\\
C = \{(1, y, z, t)\mid 4\le y\le b, y<z\le c, z<t \le d \}
\end{cases}.$$
Dễ kiểm chứng rằng có song ánh từ $A$ vào $T_3$ nên $|A|=|T_3|=d_3$.
Xét $D=\{(1, 4, z, t)\mid 5\le z \le c, z<t\le d \}$. Dễ kiểm chứng rằng $D\subset C$ và có song ánh từ $D$ vào $B$ nên $|D|=|B|$.
Ta có $A\cup B=\{(1, 3, z, t)\mid 4\le z \le c, z<t\le d \}$. Dễ kiểm chứng rằng có song ánh từ $A\cup B$ vào $T_2$ nên $|A\cup B|=|T_2|=d_2$. Chú ý rằng $A\cap B=\varnothing$ nên $|A|+|B|=d_2$ hay $|B|=d_2-d_3$. Từ đó ta có:
$$d_1=|A|+|B|+|C|\ge |A|+|B|+|D|=d_3+2|B|$$
Vậy $d_1\ge d_3+2(d_2-d_3)=2d_2-d_3$. Đẳng thức xảy ra khi và chỉ khi $b=4$.

Bài 7.

Trước hết ta chứng minh bổ đề sau: Xét một tập con $S$ bất kỳ của tập các máy tính $X$, khi đó tồn tại $1$ máy tính của hệ thống kết nối trực tiếp với ít nhất $30\%$ máy tính của $S$. \medskip

Thật vậy, xét các cặp $(s, x)$ với $s\in S,x\in X$ và $(s,x)$ kết nối trực tiếp với nhau. Khi đó, nếu tính theo $s$ thì số cặp như vậy sẽ không ít hơn $\dfrac{3}{10}|S||X|$. Do đó nếu tính theo $x$ thì sẽ phải tồn tại máy tính $x$ kết nối trực tiếp với ít nhất $\dfrac{3}{10}|S|$. \medskip

Quay trở lại bài toán, \medskip

Giả sử hệ thống có $n$ máy tính. Xét máy tính $A$ bất kỳ. Gọi $S$ là tập hợp các máy tính không kết nối trực tiếp với $A$. Nếu $S=\varnothing$ thì kết quả bài toán là hiển nhiên. Nếu $S\ne \varnothing$ thì theo bổ đề, tồn tại máy tính $B$ kết nối trực tiếp với ít nhất $30\%$ máy tính trong $S$. Ta chứng minh hai máy tính $A$ và $B$ thỏa mãn yêu cầu bài toán. \medskip

Thật vậy, giả sử $A$ kết nối trực tiếp với $k$ máy tính khác. Khi đó, theo cách chọn, $A$ và $B$ sẽ kết nối trực tiếp với ít nhất
$$k + 0,3(n-k) = 0,7k + 0,3n \ge 0,7\cdot 0,3n + 0,3n = 0,51n.$$
Từ đây ta có được kết luận của bài toán.

Bài 8.

(a) Ta sẽ chứng minh rằng $AD\perp BC$. Gọi $X$ là điểm đồng quy của $EF,MN,BC$. Do $AE,AF$ tiếp xúc với $(I)$ nên $EF$ là đường đối cực của $A$ đối với $(I)$. Ta có $X\in EF$ nên theo định lý La Hire, điểm $A$ sẽ nằm trên đường đối cực của $X$ đối với đường tròn $(I)$. \medskip

Lại có $P$ là giao điểm của $EN,FM$ nên $P$ nằm trên đường đối cực của $X$ đối với $(I)$. Vì thế nên $AP$ là đường đối cực của $X$ đối với $(I)$ hay $AP\perp BC$. Do đó $$\angle ADI=\angle AEI=\angle AFI=90^\circ.$$
Vậy $A,D,E,F$ cùng thuộc một đường tròn.

(b) Gọi $S$ là giao điểm của $BN,CM$. Xét hai tam giác $PEF,SBC$ có $PE$ cắt $SB$ tại $N$, $PF$ cắt $SC$ tại $M$, $EF$ cắt $BC$ tại $X$ và $X,M,N$ thẳng hàng. Theo định lý Desargues thì $PS,EB,FC$ đồng quy. Mặt khác $EB$ cắt $FC$ tại $A$ nên $A,P,S$ thẳng hàng, dẫn đến $S\in AD$. \medskip

Tiếp theo ta sẽ chứng minh rằng $\angle BAK=\angle CAH$. Áp dụng định lý Ceva dạng lượng giác cho tam giác $ABC$ với:

Các đường thẳng $AD,BH,CK$ đồng quy:
$$\frac{\sin\angle DAB}{\sin \angle DAC}\cdot \frac{\sin\angle HBC}{\sin \angle HBA}\cdot \frac{\sin\angle KCA}{\sin \angle KCB}=1$$
Các đường thẳng $AH,BH,CH$ đồng quy:
$$\frac{\sin\angle HAB}{\sin \angle HAC}\cdot \frac{\sin\angle HBC}{\sin \angle HBA}\cdot \frac{\sin\angle HCA}{\sin \angle HCB}=1$$
Các đường thẳng $AK,BK,CK$ đồng quy:
$$\frac{\sin\angle KAB}{\sin \angle KAC}\cdot \frac{\sin\angle KBC}{\sin \angle KBA}\cdot \frac{\sin\angle KCA}{\sin \angle KCB}=1$$

Chú ý rằng do các góc vuông và góc bù nhau nên ta có
$$\frac{\sin\angle HAC}{\sin \angle HAB}=\frac{\sin\angle KAB}{\sin \angle KAC}$$
Từ đó sử dụng công thức cộng cho mẫu thức và biến đổi thì:
$$\tan\angle HAC=\tan\angle KAB$$
Dẫn đến $\angle HAC=\angle KAB$. Cuối cùng, ta sẽ chứng minh $TB=TC$.

Gọi $U,V$ lần lượt là trung điểm của các đoạn $AK,AH$. Ta có:
$$UB=\dfrac{AK}{2}=VT,UT=\dfrac{AH}{2}=VC.$$
Đồng thời, ta cũng có:
$$\angle BUT=\angle BUA-\angle AUT=\angle AVC-\angle AVT=\angle TVC$$
Do đó $\Delta BUT=\Delta TVC$ (c.g.c), vậy nên $TB=TC$.

Đề thi và đáp án kì thi chọn đội tuyển thi Quốc gia trường Phổ thông Năng khiếu năm học 2015 -2016

Đề bài

Ngày thi thứ nhất

Bài 1.  Cho tập hợp:
$$A=\left\{ n\in \mathbb{N}\mid 1\le n\le 2015,\gcd (n,2016)=1 \right\}.$$
Hỏi có bao nhiêu số nguyên $a\in A$ sao cho tồn tại số nguyên $b$ mà $a+2016b$ là số chính phương?
Bài 2. Cho $a,b,c,d$ là các số thực thỏa mãn điều kiện:
$$\left\{\begin{array}{l}
a^2\le 1\\
a^2+b^2\le 5\\
a^2+b^2+c^2\le 14\\
a^2+b^2+c^2+d^2\le 30
\end{array} \right..$$

a)Chứng minh rằng $a+b+c+d\le 10$.
b) Chứng minh rằng $ad+bc\le 10$.

Bài 3.  Tìm tất cả các hàm số $f:\mathbb{R}\to \mathbb{R}$ thỏa mãn:
$$f\left( x-2f(y) \right)=5f(x)-4x-2f(y), \, \ \forall x,y\in \mathbb R.$$
Bài 4. Cho đường tròn $k$ và các điểm $B,C$ thuộc đường tròn sao cho $BC$ không phải là đường kính; $I$ là trung điểm $BC$. Điểm $A$ di động trên cung lớn $BC$ của $k$. Gọi $(\mathcal I_1)$ là đường tròn qua $I$ và tiếp xúc với $AB$ tại $B$, $(\mathcal I_2)$ là đường tròn qua $I$ và tiếp xúc với $AC$ tại $C$. Các đường tròn $(\mathcal I_1), (\mathcal I_2)$ cắt nhau tại $D$.

a) Chứng minh rằng đường tròn ngoại tiếp tam giác $AID$ luôn đi qua một điểm cố định.
b) Gọi $K$ là trung điểm $AD$, $E$ là tâm đường tròn qua $K$ và tiếp xúc với $AB$ tại $A$, $F$ là tâm đường tròn qua $K$ và tiếp xúc với $AC$ tại $A$. Chứng minh rằng góc $\angle EAF$ có số đo không đổi.

Ngày thi thứ hai

Bài 5. Cho dãy số $(x_n)$ được xác định bởi $x_n=\dfrac{1}{n\cos \frac{1}{n}}\ \forall n\in \mathbb N^*$. Tính giới hạn:
$$\lim_{n \to +\infty} \frac{{{x}_{1}}+{{x}_{3}}+{{x}_{5}}+\cdots+{{x}_{2n-1}}}{{{x}_{2}}+{{x}_{4}}+{{x}_{6}}+\cdots +{{x}_{2n}}}.$$
Bài 6. Tìm các giá trị của $b$ sao cho tồn tại $a$ để hệ phương trình sau có nghiệm $(x,y)$:
$$\left\{\begin{array}{l}
(x-1)^2+(y+1)^2=b \\
y=x^2+(2a+1)x+a^2
\end{array} \right..$$
Bài 7. Cho $n$ là số nguyên dương, $n\ge 2$ và $X=\left\{ 1,2,3,\ldots,n \right\}$. Gọi ${{A}_{1}},{{A}_{2}},\ldots,{{A}_{m}}$ và ${{B}_{1}},{{B}_{2}},\ldots,{{B}_{m}}$ là hai dãy các tập con khác rỗng của $X$ thỏa mãn điều kiện sau: với mỗi $i,j\in \left\{ 1,2,3,\ldots,m \right\}$, ${{A}_{i}}\cap {{B}_{j}}=\varnothing $ nếu và chỉ nếu $i=j.$
a) Chứng minh rằng với mỗi hoán vị $(x_1,x_2,\ldots,x_n)$ của tập hợp $X$, có không quá một cặp tập hợp $(A_i,B_i)$ với $i=1,2,3,\ldots,m$ sao cho nếu $x_k\in A_i$ và $x_l\in B_i$ thì ta phải có $k<l$.
b) Gọi $a_i,b_i$ lần lượt là số phần tử của tập hợp $A_i,B_i$ với $i=1,2,3,\ldots,m$. Chứng minh rằng:
$$\sum_{i=1}^{m}{\dfrac{1}{C_{{a_i+b_i}}^{a_i}}}\le 1.$$

Bài 8.  Cho tam giác $ABC$ nhọn nội tiếp đường tròn tâm $O$. Đường tròn tâm $I$ đi qua $B,C$ lần lượt cắt các tia $BA,CA$ tại $E,F.$

a) Giả sử các tia $BF,CE$ cắt nhau tại $D$ và $T$ là tâm đường tròn $(AEF)$. Chứng minh rằng $OT\parallel ID.$
b) Trên $BF,CE$ lần lượt lấy các điểm $G,H$ sao cho $AG\perp CE,AH\perp BF.$ Các đường tròn $(ABF),(ACE)$ cắt $BC$ tại $M,N$ khác $B,C$ và cắt $EF$ tại $P,Q$ khác $E,F$. Gọi $K$ là giao điểm của $MP,NQ$. Chứng minh rằng $DK\perp GH$.

Hết

Giải

Bài 1.

Cho số nguyên dương $n>1$, ta quy ước gọi một số nguyên dương $a < n$ là thặng dư chính phương theo modulo $n$ nếu $\gcd(a,n)=1$ và tồn tại số nguyên $x$ sao cho $a\equiv {{x}^{2}} \pmod n .$ \medskip

Đặt $s(n)$ là số các số như thế. Ta sẽ chứng minh hai bổ đề dưới đây: \medskip

Bổ đề 1. Cho $p$ là số nguyên tố và $k$ là số nguyên dương. Khi đó:

Nếu $p=2$ thì $s({{2}^{k}})={{2}^{\max (k-3,0)}}$.
Nếu $p>2$ thì $s({{p}^{k}})=\dfrac{{{p}^{k}}-{{p}^{k-1}}}{2}$.

Bổ đề 2. $s(n)$ là hàm nhân tính, tức là $s(ab)=s(a)s(b)$ với $\gcd(a,b)=1$. \medskip

Thật vậy, \medskip

Trước hết, ta biết rằng $s(p)=\frac{p-1}{2}$ với $p$ là số nguyên tố lẻ. Ta sẽ tính $s({{p}^{k}})$ với $k\in {{\mathbb{Z}}^{+}}$. Xét một thặng dư chính phương $a$ của $p$, khi đó tồn tại $x$ sao cho $$a\equiv {{x}^{2}}\pmod{p}.$$ Đặt $a={{x}^{2}}+pq$ thì hiển nhiên $$a\equiv {{x}^{2}}+pq \pmod {{p}^{k}}\Leftrightarrow a-pq\equiv {{x}^{2}} \pmod {{p}^{k}}$$ và khi đó, ta có ${{p}^{k-1}}$ cách chọn $q$ để các số $a-pq$ là các thặng dư chính phương theo modulo ${{p}^{k}}$. Suy ra
$$s({{p}^{k}})={{p}^{k-1}}s(p)=\frac{{{p}^{k}}-{{p}^{k-1}}}{2}.$$ Xét số nguyên tố $p=2$, với $k=1,2,3,$ dễ dàng kiểm tra được $s({{2}^{k}})=1$. \medskip

Ta xét $k\ge 4$, tương tự trên, ở bước chọn $q$, ta chỉ có 2 cách nên $s({{2}^{k}})=2s({{2}^{k-1}})$. Từ đó bằng quy nạp, ta có được $$s({{2}^{k}})={{2}^{k-3}},k\ge 4.$$ Tiếp theo, xét hai số $a,b$ nguyên dương nguyên tố cùng nhau. Gọi $A$ là tập hợp các thặng dư chính phương theo modulo $ab$ và $B$ là tập hợp các số là thặng dư chính phương chung của $a,b.$ \medskip

Nếu $x\in A$ thì tồn tại $y$ sao cho $x\equiv {{y}^{2}} \pmod{ab}$. Rõ ràng khi đó,
$$x\equiv {{y}^{2}}\pmod a, \, x\equiv {{y}^{2}}\pmod b$$
(chú ý rằng nếu $x>a$, ta có thể chọn ${x}'$ sao cho ${x}'<a$ và $x\equiv {x}'\pmod a$; tương tự với $b$).
Do đó, $x\in B$, tức là $x\in A\Rightarrow x\in B$ nên $\left| A \right|\le \left| B \right|$. \medskip

Tiếp theo, xét $x\in B$. Khi đó tồn tại $r,s$ sao cho
$x\equiv {{r}^{2}}\pmod a,\text{ }x\equiv {{s}^{2}}\pmod b$.
Theo định lý thặng dư Trung Hoa, tồn tại số nguyên $z$ sao cho $$z\equiv r\pmod a, \, z\equiv s\pmod b.$$ Khi đó $$x\equiv {{z}^{2}}\pmod a, \, x\equiv {{z}^{2}}\pmod b$$ nên
$$x\equiv {{z}^{2}} \pmod{ab}.$$ Do đó: $x\in A$, tức là $x\in B\Rightarrow x\in A$ nên $\left| A \right|\ge \left| B \right|$. Từ đây ta có $$\left| A \right|=\left| B \right| \text{ hay } s(a)s(b)=s(ab).$$ Vậy $s(n)$ là hàm nhân tính. \medskip

Các bổ đề đều được chứng minh. \medskip

Trở lại bài toán, ta thấy rằng $2016={{2}^{5}}\cdot {{3}^{2}}\cdot 7.$ Rõ ràng bài toán yêu cầu đếm số thặng dư chính phương theo modulo $2016$.
Theo bổ đề 2 thì $$s(2016)=s({{2}^{5}})s({{3}^{2}})s(7).$$ Theo bổ đề 1 thì $$s({{2}^{5}})={{2}^{2}}=4,s({{3}^{2}})=\frac{{{3}^{2}}-3}{2}=3,s(7)=\frac{7-1}{2}=3.$$ Do đó, số các số $a$ cần tìm là $4\cdot 3\cdot 3=36.$

Bài 2. 

(a) Dự đoán dấu bằng xảy ra khi $a=1,b=2,c=3,d=4$ nên ta có các đánh giá sau $
{{a}^{2}}+1\ge 2a
{{b}^{2}}+4\ge 4b
{{c}^{2}}+9\ge 6c
{{d}^{2}}+16\ge 8d
$

Do đó, ta có
$24(a+b+c+d)\le 3({{d}^{2}}+16)+4({{c}^{2}}+9)+6({{b}^{2}}+4)+12({{a}^{2}}+1)$
$=3{{d}^{2}}+4{{c}^{2}}+6{{b}^{2}}+12{{a}^{2}}+120 $

$=3({{a}^{2}}+{{b}^{2}}+{{c}^{2}}+{{d}^{2}})+({{a}^{2}}+{{b}^{2}}+{{c}^{2}})+2({{a}^{2}}+{{b}^{2}})+6{{a}^{2}}+120$

$\le 3\cdot 30+14+2\cdot 5+6\cdot 1+120=240$
Suy ra $a+b+c+d\le 10.$ \medskip

(b) Ta có $$16{{a}^{2}}+{{d}^{2}}\ge 8ad \text{ và } 9{{b}^{2}}+4{{c}^{2}}\ge 12bc.$$

Từ đó suy ra

$24(ad+bc)\le 3(16{{a}^{2}}+{{d}^{2}})+2(9{{b}^{2}}+4{{c}^{2}})$
$=3({{a}^{2}}+{{b}^{2}}+{{c}^{2}}+{{d}^{2}})+5({{a}^{2}}+{{b}^{2}}+{{c}^{2}})+10({{a}^{2}}+{{b}^{2}})+30{{a}^{2}}$
$\le 3\cdot 30+5\cdot 14+10\cdot 5+30\cdot 1=240$
Suy ra $ad+bc\le 10.$

Bài 3.

Gọi () là điều kiện đề bài cho.
Trong (
) thay $x=y=0$, ta có $$f(-2f(0))=3f(0).$$ Đặt $f(0)=a$ thì $f(-2a)=3a$.
Trong () thay $x=0$ và $y=-2a$, ta có $$f(-2f(-2a))=5a-2f(-2a)\Leftrightarrow f(-6a)=-a.$$ Trong (), thay $x=-2a,y=-6a$, ta có
$$\begin{aligned}
& f(-2a-2f(-6a))=5f(-2a)-4x-2f(-6a) \\
& \Leftrightarrow f(0)=15a+8a+2a.
\end{aligned}$$ Từ đây ta có $a=25a$ nên $a=0,$ tức là $f(0)=0$. \medskip

Trong $(*),$ thay $y=0$, ta có $$f(x)=5f(x)-4x\Leftrightarrow f(x)=x.$$ Thử lại ta thấy thỏa. Vậy hàm số cần tìm chính là $f(x)=x,\forall x\in \mathbb{R}.$

Bài 4.

(a) Gọi $O$ là tâm của đường tròn $k.$ Không mất tính tổng quát, giả sử tia $AD$ nằm giữa hai tia $AO,AB,$ các trường hợp còn lại tương tự.

Ta có: $$\angle IDB=\angle ABC,\angle IDC=\angle ACB$$ nên $$\angle BAC+\angle BDC=\angle BAC+\angle ABC+\angle ACB=180{}^\circ .$$ Do đó, tứ giác $ABDC$ nội tiếp hay $D\in (O).$ Ta thấy $$\begin{aligned}
& \angle DAO+\angle OID \\
& =\angle BAC-(\angle DAB+\angle OAC)+360{}^\circ -(90{}^\circ +\angle DIC) \\
& =\angle BAC-\left( \angle ICD+90{}^\circ -\angle ABC \right)+270{}^\circ -\angle DIC \\
& =\angle BAC+\angle ABC-(\angle ICD+\angle DIC)+180{}^\circ \\
& =(180{}^\circ -\angle ACB)-\left( 180{}^\circ -\angle IDC \right)+180{}^\circ \\
& =\angle IDC-\angle ACB+180{}^\circ =180{}^\circ.
\end{aligned} $$

Do đó, $AOID$ nội tiếp hay đường tròn $(AID)$ đi qua $O$ cố định. \medskip

(b) Ta có: $$\angle EAC=90{}^\circ -\angle BAC,\angle FAB=90{}^\circ -\angle BAC$$ nên
$$\angle EAF=180{}^\circ -2\angle BAC+\angle BAC=180{}^\circ -\angle BAC.$$

Do đó, góc $\angle EAF$ có số đo không đổi.

Bài 5.

Trước hết, ta chứng minh bổ đề sau: \medskip

Bổ đề. Giá trị của biểu thức $\frac{1}{1}+\frac{1}{2}+\frac{1}{3}+\cdots+\frac{1}{n}$ tiến tới vô cực khi $n\to +\infty.$ \medskip

Thật vậy, xét hàm số $f(x)=\ln (1+x)-x$ với $x>0$. Ta có $$f'(x)=\frac{1}{1+x}-1<0$$ nên đây là hàm nghịch biến, suy ra $f(x)<f(0)=0$ hay $\ln (1+x)<x,\forall x>0$. Thay $x$ bởi $\frac{1}{n}$, ta được
$$\ln \left( 1+\frac{1}{n} \right)<\frac{1}{n}\Leftrightarrow \frac{1}{n}>\ln (1+n)-\ln n.$$ Do đó, $$\frac{1}{1}+\frac{1}{2}+\frac{1}{3}++\frac{1}{n}>\ln 2-\ln1+\ln3-\ln2+\cdots+ln(n+1)-\ln n=\ln (n+1).$$ Vì $\ln (n+1)\to +\infty $ khi $n\to +\infty $ nên $$\frac{1}{1}+\frac{1}{2}+\frac{1}{3}+\cdots+\frac{1}{n}\to +\infty.$$
Trở lại bài toán, đặt $$y_n=\frac{x_1+x_3+x_5+\cdots+x_{2n-1}}{x_2+x_4+x_6+\cdots+x_{2n}}$$ với $n\ge 1.$

Ta thấy vì $\frac{1}{n}\in \left( 0;\frac{\pi }{2} \right)$ nên $\cos \frac{1}{n}>0$, suy ra $$x_n=\frac{1}{n\cos \frac{1}{n}}>0,n\ge 1. $$
Xét hàm số $f(t)=\frac{t}{\cos t}$ với $t\in \left( 0;\frac{\pi }{2} \right)$ thì ${f}'(t)=\frac{\cos t+t\sin t}{{{\cos }^{2}}t}>0$ nên đây là hàm đồng biến. Chú ý rằng $x_n=f\left( \frac{1}{n} \right)$, mà $\frac{1}{n}$ là dãy giảm nên $x_n$ cũng là dãy giảm. \medskip

Suy ra $x_1>x_2,x_3>x_4,\ldots,x_{2n-1}>x_{2n}$ nên $y_n>1$. \medskip

Ngoài ra, ta cũng có $x_3<x_2,x_5<x_4,\ldots,x_{2n-1}<x_{2n-2}$ nên $y_n< \frac{x_1+x_2+x_4+\cdots+x_{2n-2}}{x_2+x_4+\cdots+x_{2n}}$

$= 1-\frac{x_1-x_{2n}}{x_2+x_4+\cdots+x_{2n}}<1-\frac{x_1}{x_2+x_4+\cdots+x_{2n}}$

Dễ thấy rằng $$x_2+x_4+\cdots+x_{2n}=\sum\limits_{i=1}^{n}{\frac{1}{2i\cos \frac{1}{2i}}}\ge \sum\limits_{i=1}^{n}{\frac{1}{2i}}=\frac{1}{2}\sum\limits_{i=1}^{n}{\frac{1}{i}}.$$

Theo bổ đề trên thì $\sum\limits_{i=1}^{n}{\frac{1}{i}}$ tiến tới vô cực nên $$\lim \left( x_2+x_4+\cdots+x_{2n} \right)=+\infty .$$

Do đó $$\lim \left( 1-\frac{x_{1}}{x_2+x_4+\cdots+x_{2n}} \right)=1-0=1.$$ Theo nguyên lý kẹp, ta có $\lim x_n=1.$

Bài 6.

Đặt $X=x-1,Y=y+1$, thay vào, ta có
$$\begin{aligned}
& \left\{ \begin{aligned}
& {{X}^{2}}+{{Y}^{2}}=b \\
& Y-1={{(X+1)}^{2}}+(2a+1)(X+1)+{{a}^{2}} \\
\end{aligned} \right. \
& \Leftrightarrow \left\{ \begin{aligned}
& {{X}^{2}}+{{Y}^{2}}=b \\
& Y={{X}^{2}}+(2a+3)X+{{a}^{2}}+2a+3.
\end{aligned} \right. \\
\end{aligned}$$
Ta đưa về tìm điều kiện của $b$ để tồn tại $a$ mà hệ trên có nghiệm $(X,Y).$ Do $$Y-(X+2)={{X}^{2}}+2(a+1)X+{{(a+1)}^{2}}={{\left( X+a+1 \right)}^{2}}\ge 0$$ nên $Y\ge X+2$. Suy ra $Y-X\ge 2>0$, tức là ${{(X-Y)}^{2}}\ge 4.$ Ta có
$$b={{X}^{2}}+{{Y}^{2}}=\frac{{{(X-Y)}^{2}}+{{(X+Y)}^{2}}}{2}\ge \frac{{{(Y-X)}^{2}}}{2}\ge 2.$$ Mặt khác, với $b\ge 2$, nếu chọn $X=-(a+1)$ thì có $Y=X+2=1-a$. Khi đó, ta có
$${{X}^{2}}+{{Y}^{2}}={{(a+1)}^{2}}+{{(a-1)}^{2}}=2({{a}^{2}}+1)=b.$$ Như thế, với $a$ thỏa mãn $2({{a}^{2}}+1)=b$ thì hệ có nghiệm là $$(X,Y)=(-a-1,1-a).$$ Dễ dàng thấy rằng do $b\ge 2$ nên luôn tồn tại $a$ như thế. \medskip

Vậy các giá trị cần tìm của $b$ là $b\ge 2$.

Bài 7.

(a) Giả sử ngược lại, tồn tại $2$ cặp $(A_i,B_i)$ và $(A_j,B_j)$ thỏa mãn điều kiện đề bài đã cho. \medskip

Vì $i\ne j$ nên theo giả thiết, $$\left| A_i \cap B_j \right|\ge 1,\left| A_j\cap B_i \right|\ge 1.$$ Đặt $x_r\in A_i\cap B_j,x_s\in A_j\cap B_i$ với $1\le r,s\le n$ thì:

Do $x_r\in B_j$ nên với mọi $x_k\in A_j$, ta đều có $k<r.$
Do $x_r\in A_i$ nên với mọi $x_k\in B_i$, ta đều có $k>r$.

Từ đây suy ra $$A_j \subset \left\{ x_1,x_2,\ldots,x_{r-1} \right\},B_i\subset \left\{x_{r+1},x_{r+2},\ldots,x_n \right\}.$$

Điều này cho thấy $A_j\cap B_i=\varnothing $, mâu thuẫn với giả thiết. Vậy tồn tại không quá $1$ cặp $(A_i,B_i)$ thỏa mãn điều kiện đã cho. \medskip

(b) Gọi $T$ là tập hợp các cách chọn hai dãy $$A_1,A_2,\ldots,A_m \text{và} B_1,B_2,\ldots,B_m$$ thỏa mãn điều kiện là: với mỗi $i,j\in \left\{ 1,2,3,\ldots,n \right\}$, $A_i\cap B_j=\varnothing $ nếu và chỉ nếu $i=j.$ \medskip

Gọi $T_i\subset T$ là các cách chọn sao cho sao cho cặp $(A_i,B_i)$ thỏa mãn điều kiện là: cặp $(A_i,B_i)$ với $i=1,2,3,\ldots,n$ sao cho nếu $x_k \in A_i$ và $x_l\in B_i$ thì $x_k<x_l$ (ở đây ta xét thứ tự ban đầu của các phần tử của $X$). \hfill (*) \medskip

Theo câu (a) thì $T_i \cap T_j=\varnothing $ với $i\ne j$ nên ta có $$\left| T_1 \right|+\left| T_2 \right|+\cdots +\left| T_m \right|=\left| T_1 \cup T_2 \cup \ldots \cup T_m \right|\le T.$$ Tiếp theo, với $1\le i\le m$, xét một tập hợp $S\subset X$ và $\left| S \right|=a_i+b_i$. Khi đó, tương ứng với $S$, có đúng $1$ cách chọn $(A_i,B_i)$ thỏa mãn tính chất $(*)$ – tức là $A_i$ sẽ nhận $a_i$ số nhỏ nhất trong tập $S,$ $B_i$ là lấy phần còn lại. \medskip

Trong khi đó, nếu không có điều kiện $(*),$ ta có thể chọn tùy ý $C_{a_i+b_i}^{a_i}$ phần tử trong $S$ và $A$ và số còn lại cho $B.$ \medskip

Do đó, ta có
$\left| T_i \right|=\frac{\left| T \right|}{C_{a_i+b_i}^{a_i}} $ với $i=1,2,\ldots,m.$

Từ đây suy ra

$$\sum\limits_{i=1}^{m}\frac{\left| T \right|}{C_{a_i+b_i}^{a_i}}\le \left| T \right|\Leftrightarrow \sum\limits_{i=1}^{m}\frac{1}{C_{a_i+b_i}^{a_i}}\le 1$$

Ta có đpcm.

Bài 8. 

(a) Giả sử $EF$ cắt $BC$ ở $L$ và $(T),(O)$ cắt nhau tại $J$ khác $A.$ Suy ra $AJ$ chính là trục đẳng phương của $(T),(O).$ Do đó $OT\bot AJ$. \medskip

Khi đó,
[LB\cdot LC=LE\cdot LF] nên $L$ thuộc trục đẳng phương của $(T),(O)$. Suy ra $A,J,L$ thẳng hàng. Theo định lý Brocard cho tứ giác $BEFC$ nội tiếp trong đường tròn $(I)$ thì $I$ chính là trực tâm của tam giác $ADL.$ \medskip

Vì thế nên $ID\bot AL$, mà $OT\bot AJ$ nên $ID\parallel OT$. \medskip

(b) Dễ dàng thấy rằng $D$ là trực tâm của tam giác $AGH$ nên $AD\bot GH$. Ta sẽ chứng minh rằng $A,D,K$ thẳng hàng. \medskip

Ta có $DB\cdot DF=DE\cdot DC$ nên $D$ có cùng phương tích tới $(ABF),(AEC)$. Suy ra $AD$ chính là trục đẳng phương của $2$ đường tròn này. \medskip

Bằng biến đổi các góc nội tiếp, ta thấy rằng
$$\angle MPQ=\angle MBF=\angle CEF=\angle CNQ.$$ Suy ra $MNPQ$ nội tiếp, dẫn đến $KM\cdot KP=KN\cdot KQ$, tức là $K$ cũng có cùng phương tích tới $2$ đường tròn $(ABF),(AEC)$. \medskip

Từ đó suy ra $A,D,K$ thẳng hàng. Do đó, $DK$ vuông góc với $GH.$

Đáp án thi chọn đội tuyển trường PTNK năm 2014

Đề bài

Ngày thi thứ nhất

Bài 1. Cho $a,b,c > 0$ thỏa mãn điều kiện $(a+1)(b+1)(c+1)=1+4abc$.
Chứng mình rằng ta có bất đẳng thức $a+b+c\le 1+abc.$
Bài 2. Cho tập hợp $A=\left\{ {{n}^{3}}-4n+15|n\in \mathbb{N} \right\}.$ Tìm tất cả các phần tử $a\in A$ thỏa mãn đồng thời hai điều kiện sau đây:

i) $a$ là số chẵn.
ii) Nếu $a_1,a_2$ là các ước số của $\dfrac{a}{2}$ với $a_1,a_2>1$ thì $\gcd (a_1,a_2)>1$.

Bài 3. Tìm tất cả các hàm số $f:\mathbb N^* \rightarrow \mathbb N^*$ thỏa mãn:
$$f \left(\dfrac{f(n)}{n} \right)=n^2 \ \forall n\in \mathbb N^*.$$
Bài 4. Cho tam giác $ABC$ nội tiếp $(O)$, có $B,C$ cố định và $A$ thay đổi trên $(O).$ Ký hiệu $(I)$ là đường tròn nội tiếp tam giác $ABC.$ Gọi $({{O}_{1}})$ là đường tròn qua $A,B$ và tiếp xúc với đường tròn $(I)$ tại $E.$ Gọi $({{O}_{2}})$ là đường tròn qua $A,C$ và tiếp xúc với đường tròn $(I)$ tại $F$. Đường phân giác trong của góc $\angle AEB$ cắt $({{O}_{1}})$ tại $M$ và đường phân giác trong của góc $\angle AFC$ cắt $({{O}_{2}})$ tại $N.$

a) Chứng minh rằng tứ giác $EFMN$ nội tiếp.
b) Gọi $J$ là giao điểm của $EM$ và $FN$. Chứng minh rằng đường thẳng $IJ$ luôn đi qua một điểm cố định.

Ngày thi thứ hai

Bài 5. Cho dãy số $({{x}_{n}})$ bởi $x_0=1,x_1=2014$ và $x_{n+1}=\sqrt[3]{x_nx_{n-1}^2}\ \forall n\in \mathbb{N}^*.$

a) Chứng minh rằng dãy số $(x_n)$ có giới hạn hữu hạn và tìm giới hạn đó.
b) Với mỗi $n\ge 2$, hãy tìm số nguyên dương $k$ nhỏ nhất sao cho $a=x_n^k$ là một số nguyên. Chứng minh rằng khi đó $a$ không thể viết được dưới dạng tổng các lũy thừa bậc ba của hai số tự nhiên.

Bài 6. Cho $X$ là tập hợp gồm $19$ phần tử.

a) Chứng minh rằng tồn tại ít nhất $2600$ tập con $7$ phần tử của $X$ sao cho với hai tập con $A,B$ bất kỳ trong số $2600$ tập con đó, ta có $\left| A \cap B \right|\le 5.$
b)  Xét một họ $\Omega $ gồm $k$ tập con có $7$ phần tử của $X$. Một tập $A\subset X$ được gọi là một cận trên của $\Omega $ nếu như $\left| A \right|=8$ và tồn tại một tập con $F$ của họ $\Omega $ sao cho $F\subset A$. Gọi $d$ là số tập con cận trên của họ $\Omega$. Chứng minh rằng $d\ge \frac{3}{2}k.$

Bài 7. Cho tam giác $ABC$ không cân. Gọi $I$ là trung điểm $BC$. Đường tròn $(I)$ tâm $I$ đi qua $A$ cắt $AB,AC$ lần lượt tại $M,N.$ Giả sử $MI,NI$ cắt $(I)$ tại $P,Q$. Gọi $K$ là giao điểm của $PQ$ với tiếp tuyến tại $A$ của $(I)$. Chứng minh rằng $K$ thuộc đường thẳng $BC.$
Bài 8. Tìm số nguyên dương $n$ lớn nhất thỏa mãn các điều kiện sau:

a) $n$ không chia hết cho $3$.
b) Bảng vuông $n\times n$ không thể được phủ kín bằng $1$ quân tetramino $1\times 4$ và các quân trimino $1\times 3$. Trong phép phủ, các quân tetramino và trimino được phép quay dọc nhưng không được phép chườm lên nhau hoặc nằm ra ngoài bảng vuông.

Hết

Lời giải

Bài 1.

Điều kiện đã cho viết thành $ab+bc+ca+a+b+c=3abc$.
Chia hai vế cho $abc$ rồi đặt $a=\frac{1}{x},b=\frac{1}{y},c=\frac{1}{z}$, ta có
$xy+yz+zx+x+y+z=3.$ \medskip

Bất đẳng thức đã cho có thể viết thành
$$xy+yz+zx-xyz\le 1 \text{ hay } x+y+z+xyz\ge 2. $$
Theo bất đẳng thức Schur thì $${{(x+y+z)}^{3}}+9xyz\ge 4(xy+yz+zx)(x+y+z). $$
Đặt $m=x+y+z,n=xy+yz+zx$ thì $m+n=3$ và
$$xyz\ge \frac{4mn-{{m}^{3}}}{9}. $$
Ta sẽ chứng minh rằng
$$m+\frac{4mn-{{m}^{3}}}{9}\ge 2\Leftrightarrow {{m}^{3}}+4{{m}^{2}}-21m+18\le 0 $$ hay
$(m-2)({{m}^{2}}+6m-9)\le 0.$
Chú ý rằng ${{m}^{2}}\ge 3n$ nên
$${{m}^{2}}\ge 3(3-m)\Leftrightarrow {{m}^{3}}+3m\ge 9. $$
Do đó ${{m}^{2}}+6m-9\ge 0.$
Ta xét các trường hợp

Nếu $m>2$ thì $x+y+z>2$ nên hiển nhiên bất đẳng thức cần chứng minh là đúng.
Nếu $m\le 2$ thì $m-2\le 0$ nên ta cũng có $(m-2)({{m}^{2}}+6m-9)\le 0.$

Vậy trong mọi trường hợp, ta luôn có đpcm.

Bài 2.

Ta thấy rằng $a=n^3-4n+15$ chẵn nên $n^3+15$ chẵn hay $n$ lẻ. Đặt $n=2k+1$ với $k \in \mathbb{N}$. Ta có
$\begin{aligned}
a={{n}^{3}}-4n+15 & =(n+3)({{n}^{3}}-3n+15) \\
& =(2k+4)(4{{k}^{2}}-2k+3) \\
\end{aligned} $
nên $\frac{a}{2}=(k+2)(4{{k}^{2}}-2k+3)$.
Điều kiện ii) cho thấy rằng $\frac{a}{2}$ phải là lũy thừa của một số nguyên tố, vì nếu nó có hai ước nguyên tố trở lên, đặt là $p,q$ thì chọn $x=p,y=q$, ta có $x,y>1$ nhưng $\gcd (x,y)=1,$ không thỏa. \medskip

Vì $(4{{k}^{2}}-2k+3)-(k+2)=4{{k}^{2}}-3k+1>0$ với mọi $k\in \mathbb{N}$. Do đó, ta phải có $k+2|4{{k}^{2}}-2k+3$. Suy ra
$\frac{4{{k}^{2}}-2k+3}{k+2}=4k-10+\frac{23}{k+2}\in \mathbb{Z}. $
Do đó $k+2\in \{1,23\}$ vì $k+2>0.$ Ta xét các trường hợp

Nếu $k+2=1$ thì $k=-1$ hay $n=2k+1=-1<0$, không thỏa.
Nếu $k+2=23$ thì $k=21$ hay $n=43$, tính được $\frac{a}{2}=3\cdot {{5}^{2}}\cdot {{23}^{2}}$, cũng không thỏa.

Vậy không tồn tại số $a$ nào thỏa mãn.

Bài 3.

Với $n\in \mathbb{N}*$, ta thấy rằng nếu $n=1$ thì $f(f(1))=1$. \medskip

Nếu $n>1$ thì gọi $p$ là một ước nguyên tố bất kỳ của $n$. \medskip

Vì $\frac{f(n)}{n}\in \mathbb{N}*$ nên $n|f(n)$. Đặt $a={{v}_{p}}(n),b={{v}_{p}}\left( f(n) \right)$ thì trước hết, ta có $a\le b.$ \medskip

Từ $f\left( \frac{f(n)}{n} \right)={{n}^{2}}$, ta suy ra rằng $\left. \frac{f(n)}{n} \right|{{n}^{2}}$ hay $f(n)|{{n}^{3}}$, tức là $b\le 3a.$ \medskip

Trong biểu thức đã cho, thay $n\to \frac{f(n)}{n}$ thì
$f\left( \frac{f\left( \frac{f(n)}{n} \right)}{\frac{f(n)}{n}} \right)={{\left( \frac{f(n)}{n} \right)}^{2}}\Leftrightarrow f\left( \frac{{{n}^{3}}}{f(n)} \right)={{\left( \frac{f(n)}{n} \right)}^{2}}$
Do đó, ta phải có $\left. {{\left( \frac{f(n)}{n} \right)}^{2}} \right|\frac{{{n}^{3}}}{f(n)}\Leftrightarrow \left. {{f}^{3}}(n) \right|{{n}^{5}} \text{ nên } 3b \le 5a.$$ Sau đó lại tiếp tục thay $n$ trong biểu thức đã cho bởi $\frac{{{n}^{5}}}{{{f}^{3}}(n)}$ và cứ như thế, ta xây dựng được hai dãy hệ số của $a,b$ như sau
${{u}_{0}}={{v}_{0}}=1,{{u}_{1}}=3,{{v}_{1}}=1 \text{ và } $
${{u}_{k+1}}=2{{u}_{k-1}}+{{v}_{k}},{{v}_{k+1}}=2{{v}_{k-1}}+{{u}_{k}} \text{ với } k\ge 1. $
Khi đó $\frac{{{v}_{2k}}}{{{u}_{2k}}}\le \frac{b}{a}\le \frac{{{u}_{2k+1}}}{{{v}_{2k+1}}}. $
Biến đổi công thức của hai dãy, ta có ${{u}_{n+2}}=5{{u}_{n}}-4{{u}_{n-2}},{{v}_{n+2}}=5{{v}_{n}}-4{{v}_{n-2}}$ và cả hai dãy đều có phương trình đặc trưng là ${{t}^{2}}-5t+4=0$. Ngoài ra, dãy chẵn và dãy lẻ trong mỗi dãy đều độc lập với nhau.

Ta có ${{u}_{0}}=1,{{u}_{2}}=3,{{v}_{0}}=1,{{v}_{2}}=5$ nên
\[{{u}_{2k}}=\frac{13+2\cdot {{16}^{k}}}{15},{{v}_{2k}}=\frac{11+4\cdot {{16}^{k}}}{15},k\ge 1. \] Từ đó, dễ dàng tính được $\lim \frac{{{u}_{2k+1}}}{{{v}_{2k+1}}}=2$. \medskip

Một cách tương tự, ta tính được $\lim \frac{{{u}_{2k}}}{{{v}_{2k}}}=\frac{1}{2}$. Do đó, số $\frac{b}{a}$ bị kẹp ở giữa và là số nguyên nên chỉ có thể là $\frac{b}{a}=2\Leftrightarrow b=2a.$ \medskip

Rõ ràng tập hợp ước nguyên tố của $n$ và $f(n)$ là giống nhau. Hơn nữa, với một ước nguyên tố cụ thể thì số mũ trong $f(n)$ gấp đôi số mũ trong $n.$ Suy ra $f(n)={{n}^{2}}, \forall n>1.$ \medskip

Tiếp theo, giả sử $f(1)=n>1$ thì ta có $f(f(1))=1$ nên $f(n)=1,$ mâu thuẫn. Vì thế nên chỉ có thể $f(1)=1.$ \medskip

Vậy tất cả các hàm thỏa mãn là $f(n)={{n}^{2}},\forall n\in \mathbb{N}^*$.

Bài 4.

(a) Trước hết, ta thấy rằng ${{O}_{1}},I,E$ thẳng hàng và ${{O}_{2}},I,F$ thẳng hàng. \medskip

Vì $M$ là trung điểm cung $AB$ của $({{O}_{1}})$ nên ${{O}_{1}}M$ là trung trực của $AB$, suy ra $O\in {{O}_{1}}M.$ Tương tự, ta cũng có $O\in {{O}_{1}}N.$ \medskip

Gọi $P,Q$ lần lượt là tiếp điểm của $(I)$ với $AB,AC.$ \medskip

Vì $IP\parallel {{O}_{1}}M$ (cùng vuông góc với $AB$) nên $\angle M{{O}_{1}}E=\angle PIE$.
Hơn nữa, các tam giác ${{O}_{1}}ME,IPE$ đều cân với đỉnh là ${{O}_{1}},I$ nên suy ra chúng đồng dạng, tức là
$\angle IEP=\angle {{O}_{1}}EM$ hay $E,P,M$ thẳng hàng. Tương tự thì $F,Q,N$ cũng thẳng hàng. \medskip

Vì ta đã có $E,F,P,Q$ cùng thuộc đường tròn $(I)$ nên để có $E,F,M,N$ cùng thuộc một đường tròn thì $\angle EMN=\angle EFN=\angle EPQ$ hay $MN\parallel PQ.$ \medskip

Mặt khác, $AI\bot PQ$ nên ta cần có $AI\bot MN.$ \medskip

Thật vậy, sử dụng phương tích với đường tròn $(I)$ ta có
\[M{{A}^{2}}-N{{A}^{2}}=MP\cdot ME-NQ\cdot NF=M{{I}^{2}}-N{{I}^{2}} \] nên theo định lý bốn điểm thì $AI\bot MN$, từ đó ta có đpcm. \medskip

(b) Vì $PQ\parallel MN,OM\parallel IP$ nên dễ dàng có $\angle IPQ=\angle OMN$. Tương tự $\angle IPQ=\angle ONM.$ \medskip

Do đó, hai tam giác $IPQ,OMN$ đồng dạng với nhau, tức là $$\frac{IP}{OM}=\frac{PQ}{MN}.$$
Ngoài ra, $$\frac{JP}{JM}=\frac{IP}{OM},$$ kết hợp với $\angle JPI=\angle JMO$, ta có hai tam giác $JPI,JMO$ đồng dạng, dẫn đến $$\angle PJI=\angle MJO.$$

Từ đây suy ra $I,J,O$ thẳng hàng hay $IJ$ luôn đi qua điểm $O$ cố định.

Bài 5.

(a) Đặt $u_n=\log_{2014}(x_n)$ thì ta thu được dãy $(u_n)$ như sau
$\left\{\begin{matrix} u_0=0,u_1=1\\ u_{n+1}=\dfrac{1}{3}u_n+\dfrac{2}{3}u_{n-1} \end{matrix}\right. $
Từ đó tìm được
$u_n=\dfrac{3}{5}-\dfrac{3}{5} \cdot \left ( \dfrac{-2}{3} \right )^n$
Suy ra $\underset{n\rightarrow +\infty }{\lim}u_n=\dfrac{3}{5}$ nên ta có được
$\underset{n\rightarrow +\infty }{\lim} x_n=\underset{n\rightarrow +\infty }{\lim}(2014^{u_n})=2014^{3/5} $
(b) Ta thấy rằng để có $(x_n)^k$ là một số nguyên thì $\dfrac{3k(3^{n}-(-2)^n)}{5 \cdot 3^n}\in \mathbb{Z}$ nguyên.
Ta xét các trường hợp

Nếu $n$ lẻ thì $3^n-(-2)^n=3^n+2^n\;\vdots\; 5$. Vì $\gcd\left ( \dfrac{3^n+2^n}{5},3^n \right )=1$ nên ta được $3^n\mid 3k$ nên $k$ nhỏ nhất thỏa mãn điều này là $k=3^{n-1}$.
Nếu $n$ chẵn thì $3^n-2^n\equiv (-2)^n-2^n=0 \pmod{5}$ và tương tự, ta cũng tìm được $k=3^{n-1}$.

Do đó số $k$ nhỏ nhất cần tìm là $k=3^{n-1}$.
Tiếp theo, ta sẽ chứng minh rằng phương trình sau không có nghiệm tự nhiên
$a^3+b^3=2014^n \Leftrightarrow (a+b)(a^2-ab+b^2)=2014^n $

Gọi ${{n}_{0}}$ số nguyên dương nhỏ nhất sao cho tồn tại $a,b\in {{\mathbb{Z}}^{+}}$ để ${{a}^{3}}+{{b}^{3}}={{2014}^{{{n}_{0}}}}$.
Dễ thấy ${{n}_{0}}=1$ không thỏa nên ta chỉ xét ${{n}_{0}}\ge 2.$
Ta xét các trường hợp

Nếu $\gcd (a+b,{{a}^{2}}-ab+{{b}^{2}})=1$ thì dễ thấy ${{(a-b)}^{2}}\ge 1$. Khi đó
\[{{a}^{2}}-ab+{{b}^{2}}\ge a+b>\sqrt{{{a}^{2}}-ab+{{b}^{2}}}. \] Vì $2014=2\cdot 19\cdot 53$ nên chỉ có thể xảy ra \[a+b={{19}^{{{n}_{0}}}},{{a}^{2}}-ab+{{b}^{2}}={{106}^{{{n}_{0}}}}. \] Ngoài ra ${{(a+b)}^{2}}\le 4({{a}^{2}}-ab+{{b}^{2}})$ nên ta phải có ${{361}^{{{n}_{0}}}}\le 4\cdot {{106}^{{{n}_{0}}}}$. Đánh giá này sai khi ${{n}_{0}}\ge 2$ nên trường hợp này không thỏa.
Nếu $\gcd (a+b,{{a}^{2}}-ab+{{b}^{2}})>1$ thì chẳng hạn
\[a+b={{2}^{x}}u,{{a}^{2}}-ab+{{b}^{2}}={{2}^{y}}v \text{ với } \gcd (u,2)=\gcd (v,2)=1.\] Các trường hợp còn lại chứng minh tương tự.
Ngoài ra $uv={{1007}^{{{n}_{0}}}},x+y={{n}_{0}}. $
Chú ý rằng ${{(a+b)}^{2}}-({{a}^{2}}-ab+{{b}^{2}})=3ab$ nên $3ab$ cũng chẵn, tức là cả hai số $a,b$ đều chẵn (vì nếu không thì ${{a}^{3}}+{{b}^{3}}$ lẻ).

Từ đây dễ dàng chứng minh được $3{{v}_{2}}(a)=3{{v}_{2}}(b)={{n}_{0}}$, ta đưa về
${{{x}’}^{3}}+{{{y}’}^{3}}={{1007}^{{{n}_{0}}}}$.
Cứ như thế, ta được $2014|a,2014|b$ nên phương trình sau cũng có nghiệm nguyên dương
${{\left( \frac{a}{2014} \right)}^{3}}+{{\left( \frac{b}{2014} \right)}^{3}}={{2014}^{{{n}_{0}}-3}}. $
Điều này mâu thuẫn với các chọn ${{n}_{0}}$ nên phương trình trên vô nghiệm. Các trường hợp còn lại tương tự.
\end{enumerate}
Ta có đpcm.

Bài 6.

(a) Không mất tính tổng quát, ta có thể giả sử $X$ là tập hợp $19$ số nguyên dương đầu tiên.
Gọi $X(k)$ là tập hợp tất cả các tập con có $7$ phần tử của $X$ và tổng các phần tử của nó chia $19$ dư $k$. \medskip

Khi đó, dễ thấy rằng $\left| X(0) \right|+\left| X(1) \right|+\cdots +\left| X(18) \right|$ chính là số tập con có $7$ phần tử tùy ý của $X$ và là $C_{19}^{7}.$ \medskip

Ta thấy rằng hai tập hợp $A,B\in X(k)$ tùy ý đều thỏa mãn đề bài. \medskip

Thật vậy, \medskip

Giả sử $\left| A\cap B \right|=6$ (không thể có $\left| A\cap B \right|=7$ vì khi đó hai tập hợp trùng nhau). Đặt
$A=\{{{a}_{1}},{{a}_{2}},{{a}_{3}},{{a}_{4}},{{a}_{5}},{{a}_{6}},x\},B=\{{{a}_{1}},{{a}_{2}},{{a}_{3}},{{a}_{4}},{{a}_{5}},{{a}_{6}},y\}$
thì \[\sum\limits_{i=1}^{6}{{{a}_{i}}}+x\equiv \sum\limits_{i=1}^{6}{{{a}_{i}}}+y\equiv k\pmod{19}\] nên $x\equiv y\pmod{19}$. Suy ra $x=y$, mâu thuẫn.
Đến đây, dễ thấy rằng \[\underset{0\le k\le 18}{\mathop{\max }}\,\left\{ \left| X(k) \right| \right\}\ge \frac{C_{19}^{7}}{19}=2652>2600.\]Ta có đpcm. \medskip

(b) Xét một tập hợp $F$ thuộc họ $\Omega $. Vì $\left| X\backslash F \right|=19-7=12$ nên có tất cả $12$ tập hợp $A\subset X$ với $\left| A \right|=8$ và $F\subset A.$ \medskip

Ngược lại, ứng với một tập hợp $A$ là một cận trên của họ $\Omega $, có không quá $8$ tập $F$ trong họ $\Omega $ sao cho $F\subset A.$ Do đó $d\ge \frac{12}{8}k$ hay $d\ge \frac{3}{2}k.$ \medskip

Đẳng thức xảy ra khi họ $\Omega $ là tập hợp tất cả các tập con có $7$ phần tử của $X$.

Bài 7.

Không mất tính tổng quát, giả sử $AB<AC.$ \medskip

Kẻ đường kính $AJ$ của đường tròn $(I).$ Khi đó, dễ thấy tứ giác $ABJC$ và $ANJQ$ là các hình bình hành nên $JB\parallel AC,JQ\parallel AN$ dẫn đến $J,Q,B$ thẳng hàng. Tương tự $J,P,C$ thẳng hàng. \medskip

Gọi $H$ là hình chiếu của $A$ lên $BC$ thì tứ giác $AQBH$ nội tiếp. \medskip

Suy ra
\[\angle QHB=\angle QAB=\angle QAM=\angle QPM=\angle QPI \] nên tứ giác $PQHI$ cũng nội tiếp.
Gọi $(O)$ là đường tròn ngoại tiếp tam giác $ABC$ thì dễ thấy đường tròn $(AHI)$ tiếp xúc với $(O)$ tại $A.$

Xét ba đường tròn $(O),(AHI),(PQHI)$ thì

Trục đẳng phương của $(O),(AHI)$ là tiếp tuyến của $(O)$ tại $A$.
Trục đẳng phương của $(O),(PQHI)$ là $PQ$.
Trục đẳng phương của $(PQHI),(AHI)$ là $HI.$

Do đó, $K$ chính là tâm đẳng phương của ba đường tròn nên $K\in HI$ hay $K,B,C$ thẳng hàng.

Bài 8.

Ta sẽ chứng minh $n = 5$ là giá trị lớn nhất cần tìm. \medskip

Ta nhận thấy rằng nếu $n = 3k+1, k \ge 1$ thì ta luôn phủ được bảng vuông $n \times n$ bằng cách phủ hàng đầu tiên bằng $1$ quân tetramino kích thước $1 \times 4$ (ta sẽ gọi tắt là tetramino) và $k-1$ quân trimino kích thước $1 \times 3$ (ta sẽ gọi tắt là trimino). Các cột còn lại có chiều dài $3k$ có thể phủ được bằng các quân trimino (xoay dọc lại). \medskip

Ta chứng minh rằng nếu $n = 3k+2, k \ge 2$ thì bảng vuông $n \times n$ cũng phủ được. Cách phủ với $n = 8$ được minh họa như sau

Dễ dàng thấy rằng với $k \ge 3$ thì ta có thể thu được cách phủ cho bảng vuông $n \times n$ bằng cách phủ phần hình vuông $8 \times 8$ ở góc trên bên trái như trên, phần còn lại gồm $1 $ hình chữ nhật kích thước $3(k-2) \times (3k+2)$ và 1 hình chữ nhật kích thước $8 \times 3(k-2)$ phủ được bằng các quân trimino.

Bây giờ ta chứng minh bảng vuông $5 \times 5$ không thể phủ được bằng 1 quân tetramino và 7 quân trimino.

Trước hết ta chứng minh bổ đề: Nếu bảng vuông $5 \times 5$ có thể phủ được bằng một hình vuông $1 \times 1$, ta gọi là unomino và $8$ quân trimino thì quân unomino $1 \times 1$ phải phủ ô trung tâm. \medskip

Thật vậy, \medskip

Ta đánh số các ô của bảng vuông $5 \times 5$ như hình vẽ

Ta thấy rằng một quân trimino luôn phủ đúng một ô mang số $1,$ một ô mang số $2$ và một ô mang số $3.$ Vì số các số $2$ bằng $9,$ còn số các số $1$ và $3$ bằng $8$ nên nếu phép phủ ở đề bài thực hiện được thì quân unomino phải phủ một ô mang số $2.$ \medskip

Mặt khác, ta có thể đánh số bảng vuông $5 \times 5$ bằng một cách khác

Các tính chất nói ở trên vẫn đúng cho cách đánh số này, tuy nhiên ở đây số số $1$ là $9$, còn số số $2$ và $3$ là $8.$ Do đó, một lần nữa ta kết luận quân unomino phải phủ một ô mang số $1.$ \medskip

Giao hai điều kiện cần nói trên lại, ta thấy với một cách phủ hợp lệ thì quân unomino phải phủ ô trung tâm. \medskip

Quay trở lại với vấn đề phủ bảng vuông $5 \times 5$ bằng $1$ quân tetramino và $7$ quân trimino. Nếu tồn tại một cách phủ như thế thì cắt quân tetramino thành $1$ quân unomino và $1$ quân trimino, ta thu được một phép phủ bảng vuông $5 \times 5$ bằng $1$ quân unomino và $8$ quân trimino. \medskip

Theo bổ đề thì quân unomino phải nằm ở ô trung tâm, nghĩa là một đầu của quân tetramino phải nằm ở ô trung tâm, mâu thuẫn (vì khi đó quân tetramino sẽ bị lòi ra ngoài bảng vuông). \medskip

Với những lý luận ở trên, ta kết luận $n = 5$ là giá trị lớn nhất cần tìm.